Pytanie:
Czy wiemy, dlaczego w naszym wszechświecie obowiązuje ograniczenie prędkości?
TheQuantumMan
2016-01-20 23:07:18 UTC
view on stackexchange narkive permalink

To pytanie dotyczy tego, dlaczego mamy uniwersalne ograniczenie prędkości (prędkość światła w próżni). Czy istnieje bardziej fundamentalne prawo, które mówi nam, dlaczego tak jest?

Nie pytam, dlaczego ograniczenie prędkości jest równe $ c $ i nie coś innego, ale dlaczego w ogóle istnieje ograniczenie.

EDYCJA: Odpowiedzi typu „jeśli nie było…” i odpowiedzi wyjaśniające konsekwencje posiadania lub braku ograniczenia prędkości nie są - w moim zdaniem - udzielając odpowiedzi na pytanie, czy istnieje bardziej podstawowy sposób wyprowadzenia go z prawa wyjaśniającego ten limit.

Fizyka to nauka o znajdowaniu ogólnych zasad dotyczących tego, co widzimy we wszechświecie.Jeśli chcesz udowodnić coś, czego nie może być lub nie ma, to używanie tych reguł jako aksjomatów i przedstawianie dowodu przez sprzeczność jest tym, co ogranicza twój warunek „Odpowiedzi podobne”, jeśli nie było ... ”i odpowiedzi wyjaśniająceKonsekwencje posiadania lub nieprzestrzegania ograniczenia prędkości nie są - moim zdaniem - udzieleniem odpowiedzi Wh
34 odpowiedzi:
Kostya
2016-01-21 00:02:17 UTC
view on stackexchange narkive permalink

Wyobraź sobie, że jest osoba, która woli mierzyć kwotę pieniędzy na swoim koncie bankowym o wartości $ V $ . Równanie to $ V = C \ tanh N $ , gdzie $ N $ to rzeczywista kwota pieniędzy w dolarach. Ta osoba również będzie zdezorientowana:

Dlaczego istnieje limit ( C $ C $ ) kwoty, którą mogę mieć ? Czy istnieje prawo, które mówi, że wartość moich pieniędzy $ V $ nie może przekraczać $ C $ span >?

Odpowiedź jest taka, że ​​używa on tylko „złej” zmiennej do pomiaru swoich aktywów. $ V $ nie jest addytywny - jest to przekształcenie zmiennej addytywnej $ N $ , którą on musi używać, aby wszystko miało sens. I nie ma „prawa wszechświata”, które ogranicza wartość $ V $ - takie ograniczenie jest tylko produktem jego uporu.

To samo dotyczy pomiaru prędkości - jest to „zła” zmienna opisująca prędkość; prędkość nie jest addytywna. „Prawidłowa” zmienna to „ szybkość” - jest addytywna i nie ma ograniczeń.

Komentarze nie służą do rozszerzonej dyskusji;ta rozmowa została [przeniesiona do czatu] (http://chat.stackexchange.com/rooms/34698/discussion-on-answer-by-kostya-do-we-know-why-there-is-a-speed-limit-in-our-univ).
Nie zamierzam kłamać, to prawie nie miało dla mnie sensu.Czy możemy to trochę stłumić?
Ale DLACZEGO nasza miara prędkości powinna być określana przez funkcję asymptotyczną?
w porządku, więc teraz pytanie brzmi: dlaczego rapidity = $ \ arctan (v / c) $?dlaczego $ c $?
A dlaczego nie po prostu szybkość = $ v $?
Nie podoba mi się ta odpowiedź.Jeśli zmierzę czas potrzebny na dotarcie światła na Księżyc iz powrotem, mogę zmierzyć, że odległość / czas to „c”.To jest definicja szybkości.To jest skończona miara.Wyodrębnienie nieskończonej wartości z tych dwóch prostych pomiarów czasu trwania i odległości wydaje się w najlepszym przypadku sztuczne.
@njzk2 Powinieneś rozwinąć temat pomiaru odległości.To nie jest tak „proste”, jak mogłoby się wydawać.
Oto interaktywny wykres V = C tanh N: https://www.desmos.com/calculator/0iilcaslrn
Uważam, że ta odpowiedź właściwie dotyczyła tego, dlaczego wartości prędkości nie mogą przekroczyć naszej stałej c i dlaczego sama wartość nic nie znaczy, ale uważam, że nie wyjaśnia, dlaczego nic nie może być szybsze od światła, co moim zdaniem jest duchem pytania OP.(Mogę to wszystko źle odczytać.)
Myślę, że zamieszanie wynika z historii: co zrozumiałe, gdy po raz pierwszy próbowano obliczyć tempo podróży, funkcją najniższego rzędu opisującą wszystkie dostępne doświadczenia (w granicach błędu) była $ v = \ mathrm {odległość} / \ mathrm {czas} $ -- chociaż nie myślę o tym w tych kategoriach. Gdy sprzęt stał się bardziej zaawansowany, a badania świata bardziej szczegółowe, odkryliśmy, że dodanie $ v $ psuje się i że szybkość $ y $ jest bardziej poprawna.Ale a) większość ludzi nie zdaje sobie sprawy z $ y $ i b) obliczenie tego jest o wiele trudniejsze.Więc $ v $ jest generalnie „złe”, ale bardzo wygodne w większości zastosowań.
@JosiahKrutz Nigdy nie zdawałem sobie sprawy, że możesz użyć zmiennych innych niż $ x $ jako zmiennej niezależnej w Desmos.Dzięki!
Warto zaznaczyć, że niektóre rzeczy mogą przebiegać szybciej niż światło.Ograniczenie dotyczy materii i informacji.Zobacz „jasne plamy i cienie” tutaj: https://en.wikipedia.org/wiki/Faster-than-light
Śliczna odpowiedź, ale raczej „tani strzał”.Ukrywasz wszystkie głębokie implikacje, jakby to była tylko parametryzacja problemu.Równie dobrze mógłbym powiedzieć, że pytanie jest `` kowariantne '' z parametryzacją, więc teraz staje się `` Dlaczego szybkość nie może być złożona? '';)
Niezwykle rozświetlający.
JDługosz
2016-01-21 15:57:13 UTC
view on stackexchange narkive permalink

Jest taki wspaniały artykuł, który pamiętam, kiedy czytałem, który używa tylko podstawowej algebry tylko do określenia najbardziej ogólnej postaci wzoru na dodanie prędkości, opartej tylko na ogólnych zasadach symetrii (to, co działa tutaj , działa również tam itp.).

Nie mogę znaleźć tego, ale łatwo jest znaleźć tylko teoria względności . I inne, które pochodzą z początkowej wersji Mermina.

Kolejna praca z tym samym pomysłem (ale różnymi określonymi aksjomatami) to Jeszcze jedno wyprowadzenie transformacja Lorentza Jean-Mark Lévy-Leblond opublikowana w 1976 roku (dzięki bdforbs).

W koniec, pokazuje, że znana szczególna teoria względności jest nieuniknioną odpowiedzią. W artykule wyszła z tego dowolna nieokreślona wartość $ Z $ . Istnieją 3 przypadki: negatyw nie zadziałał (w artykule, który pamiętam, matematyka dławi się. W artykule, do którego prowadzi łącze, jest to „niespójne samoistnie”, ale Greg Egan szczegółowo opracował ten przypadek). 0 $ daje stały czas bezwzględny Galileo, a każda wartość dodatnia daje szczególną teorię względności z ograniczeniem prędkości.

Prędkość po prostu jest . W jednostkach naturalnych jest to wartość 1 $ . Powodem, dla którego prędkość światła (lub jakiejkolwiek rzeczy bezmasowej) wydaje się być czymś szczególnym, jest jej związek z innymi rzeczami. W końcu możesz stwierdzić, że zależność zwana stałą o drobnej strukturze ma szczególną wartość.

Twoje prawdziwe pytanie brzmi: dlaczego czy stała drobnej struktury jest wartością, którą jest? Odpowiedź nie jest znana. Może być zdeterminowany przez głębszy zestaw reguł, niż znamy teraz, może pochodzić z fizyki, której nie jesteśmy świadomi, lub może bądź czystym przypadkiem, jak liczba planet w naszym Układzie Słonecznym, a nie prawem.


Dlaczego więc $ Z $ jest niezerowe? Cóż, jeśli jest to dowolna wartość losowa, ma znikomo małą szansę na to, że będzie dokładnie zero, a rzeczywista wartość, o ile nie jest dokładnie zerem, po prostu ustawia współczynnik skalowania i tak naprawdę nic nie znaczy.

Bogactwo Wszechświata wynika z wyłaniających się efektów podstaw: biorąc pod uwagę, że czas jest względny w połączeniu z mechaniką kwantową oznacza, że ​​ antycząstki muszą istnieć wraz z produkcją par i unicestwieniem.

Gdyby wszechświat nie miał szczególnej teorii względności, ale miał ustalony czas absolutny, byłby bardzo inny i nie mielibyśmy w ogóle tego samego rodzaju. Wszystko do siebie pasuje i ogólnie rzecz biorąc, jeśli spojrzysz na jedną głęboką funkcję, okaże się, że jest ona koniecznie oparta na innych głębokich funkcjach. To wszystko albo nic: nie możesz wybrać samej skończonej szybkości przyczynowości, tak jak nie możesz zapytać, dlaczego jeden bok trójkąta ma określoną długość.


Uwagi

Podlinkowany artykuł i inne powiązane (z Mermin?) Użyj wartości K w nieco inny sposób niż Z w artykule, którego nie mogę teraz znaleźć. Te łatwe do znalezienia również używają rachunku różniczkowego i granic, co w tym celu nie jest tak satysfakcjonujące, jak użycie samej algebry z czterema założonymi związkami ze względu na symetrię.

jeden, który pamiętam pierwotnie używany (IIRC)

  1. przestrzeń jest taka sama tu i tam
  2. jeśli A widzi, że B porusza się z prędkością X, B widzi, że A porusza się z prędkością X w przeciwny kierunek. ekwiwal. kosmos jest taki sam we wszystkich kierunkach w jednowymiarowym modelu.
  3. wyniki eksperymentów są teraz takie same, jak wyniki eksperymentów przeprowadzonych w różnym czasie. (to samo co nr 1, ale raczej T niż X)
  4. to samo co nr 2, ale raczej T niż X

Myślę, że jest to bardziej satysfakcjonujące niż Lévy- Leblond, który przyjmuje uniwersalną zgodność przyczynowości jako aksjomat.

To jest w pewnym sensie podejście antropiczne, ale nie mniej prawdziwe.Wydaje się to sprzeczne z intuicją, ale tylko dlatego, że widzimy rzeczy na mniejszą skalę, gdzie zasady wydają się inne.To tak, jakby zapytać, jak ludzie na spodniej części ziemi mogą pozostać przywiązani, jeśli ziemia jest okrągła.To rozumowanie pochodzi z tego, co intuicyjnie rozumiemy, ale to nie znaczy, że to samo odnosi się do większej skali.
Zaczynając od rozważań na temat symetrii, możesz pokazać, że rzeczywiście masz tylko szczególną teorię względności jako możliwą odpowiedź, ale c = nieskończoność nie jest w ogóle wykluczona
@Neil To prawda, ale zasadą, którą intuicyjnie rozumiemy w tym przypadku, jest przyczynowość: trochę trudno jest zrozumieć wszechświat bez przyczyny, który powstałby, gdyby wybrano znak ujemny.Co więcej, nie jest to jedyny efekt, a wszystkie dziwne efekty, które pokazałby wszechświat Riemanna, są badane w trylogii „Ortogonalnej” Grega Egana.Egan podaje podsumowanie niektórych fizyki [tutaj] (http://gregegan.customer.netspace.net.au/ORTHOGONAL/00/PM.html).
Myślę, że wspaniałym artykułem, o którym wspominasz, jest [Jak Galileo mógł wyprowadzić specjalną teorię względności] (http://dx.doi.org/10.1119/1.17636).
@Ruslan nie, to nie ten.Fale ręczne „używają symetrii”, podczas gdy ta, którą pamiętam, polegała na wykorzystaniu 4 tożsamości (tłumaczenia i odbicia w przestrzeni i czasie), aby uzyskać tę funkcję.
Mermin tego nie wymyślił.Sięga wstecz do artykułu Ignatowskiego z 1911 roku.
Czy jest to „jeszcze jedno wyprowadzenie transformaty Lorentza”?https://doi.org/10.1119/1.10490
„Cóż, jeśli jest to jakakolwiek przypadkowa wartość, ma znikomo małą szansę na osiągnięcie dokładnie zera”. Nie jestem pewien, czy to dobry argument.„Ilość” równa zero (tj. W pewnym sensie „nieistniejąca”) w teorii wydaje się czynić ją mniej oszczędną, a zatem, według brzytwy Ockhama, bardziej prawdopodobnym a-priori.
Michael Seifert
2016-01-21 22:40:43 UTC
view on stackexchange narkive permalink

Najlepsza odpowiedź, jaką mogę wymyślić, to „ponieważ w przeciwnym razie Wszechświat byłby zasadniczo nieprzewidywalny”.

Możemy wyobrazić sobie czasoprzestrzeń jako czterowymiarową rozmaitość $ \ mathcal {M} $; prawa fizyki narzucają następnie, jak materia i energia zachowują się w tej rozmaitości. (Ze względu na argumentację można to postrzegać jako zwykłą, starą płaską przestrzeń Minkowskiego, chociaż argument uogólnia się również na zakrzywione czasoprzestrzenie.) Możemy wtedy zadać następujące pytanie: „Załóżmy, że wiem, jak zachowuje się materia i energia pewna skończona część wszechświata w pewnym momencie $ t = 0 $. Co to mówi mi o zachowaniu się materii i energii we Wszechświecie po tym czasie? "

$ \ hspace {100px} $

Jeśli we Wszechświecie istnieje ograniczenie prędkości, to istnieje obszar czasoprzestrzeni zwany domeną zależności , w której można przewidzieć, co stanie się po naszej początkowej chwili. Składa się ze wszystkich zdarzeń czasoprzestrzennych, których przeszłe stożki świetlne 1 , śledzone wstecz do $ t = 0 $, są całkowicie zawarte w obszarze przestrzeni, o którym mieliśmy wiedzę. Rozpatrywany jako funkcja czasu, obszar przestrzeni leżący w domenie zależności będzie stopniowo kurczył się do zera, gdy wpływy spoza naszego początkowego obszaru (o którym nie mieliśmy pojęcia) będą się rozprzestrzeniać do wewnątrz. Ale jeśli wszechświat ma ograniczenie prędkości we wszystkich punktach, to mamy gwarancję, że istnieje skończona objętość czasoprzestrzeni, w której możemy przewidzieć, co się stanie.

Jeśli jednak istnieje nieskończona prędkość propagacji Wszechświata, wówczas domena zależności zanika. Z grubsza mówiąc, nie ma sposobu, abyśmy mogli cokolwiek przewidzieć, ponieważ wpływy przyczynowe mogą rozprzestrzeniać się spoza naszego regionu danych początkowych i zepsuć wszystko natychmiast po $ t = 0 $. Tak więc, gdyby nie było ograniczenia prędkości, Wszechświat byłby w zasadzie nieprzewidywalny; bez wiedzy o wszystkim , co działo się we Wszechświecie w określonej chwili, prawa fizyki nie miałyby mocy predykcyjnej.

Mogę swobodnie przyznać, że nie jest to odpowiedź na pytanie „dlaczego istnieje ograniczenie prędkości?” jako "jak wyglądałby wszechświat, gdyby nie było ograniczenia prędkości?" Mimo to Wszechświat bez ograniczenia prędkości jest na tyle obcy i niezrozumiały, że cieszę się, że żyję we Wszechświecie z jednym. (Ma to odcienie argumentu antropicznego - być może w jakimś równoległym Wszechświecie jakieś niezrozumiałe istoty argumentują, jak okropne byłoby życie we Wszechświecie z ograniczeniem prędkości.)

Na koniec zauważ, że nic w tym argumencie nie opiera się na szczególnej teorii względności; wszystko, czego potrzeba, to istnienie pojęcia „stożka światła” w każdym punkcie przestrzeni. Ograniczenie prędkości może się różnić w zależności od punktu lub może różnić się w zależności od kierunku, ale tak długo, jak oddziela sąsiedztwo każdego punktu czasoprzestrzeni na przyczynową przeszłość, przyczynową przyszłość i przyczynowo niezwiązane regiony (jak w przypadku konwencjonalnych czopków świetlnych w konwencjonalnej teorii względności) , to argument nadal występuje.


1 „Stożek światła” niekoniecznie oznacza „ścieżkę wszystkich promieni świetlnych prześledzonych wstecz w czasie”, ale raczej „ścieżki wszystkich promieni podróżujących z ograniczeniem prędkości są wytyczone w czasie”.

Dopóki siła efektów spada do 0 wraz z odległością, nie widzę powodu, dla którego ta idea „domeny zależności” jest koniecznością.My ** już ** nie możemy dokładnie przewidzieć, co się stanie w dowolnej skończonej objętości przestrzeni, a nie mamy nic przeciwko temu, więc dlaczego miałoby to coś zmieniać?
@immibis: Nie ma powodu, aby oczekiwać, że siła efektów spadnie do 0. W rzeczywistości, jeśli patrzysz na wszystkie rzeczy, które mogą wpłynąć na ciebie w ciągu „jednego tyknięcia zegara”, spodziewasz się tych znajdujących się dalej (które musiałyby poruszać się szybciejby do ciebie dotrzeć), aby urosnąć kwadratowo pod względem siły i odległości.
ta odpowiedź jest głęboka.dobry.
@R .. Promieniowanie elektromagnetyczne i grawitacja, oba rozpadają się jako $ \ frac {1} {r ^ 2} $ - rozkład kwadratowy eliminuje kwadratową ilość rzeczy, które mogą wpływać na ciebie, więc rzeczy oddalone o 500 miliardów km nie mogąwpływają na ciebie nie bardziej niż na rzeczy oddalone o 5 cm.(Całkowity efekt jest nadal nieograniczony w teorii, ale w praktyce przestrzeń oddalona o 500 miliardów km jest wypełniona w znacznie mniejszym stopniu)
Zamiast cząstki.
Czy Wszechświat nie jest już zasadniczo nieprzewidywalny?Rozpad radioaktywny: nie masz pojęcia, kiedy dokładnie jeden atom ulegnie rozpadowi.Jeśli masz jeden z nich w swojej obserwowalnej części przestrzeni, nie możesz przewidzieć, jak będzie wyglądał w momencie t = 1, ponieważ mógł, ale nie musi, ulec rozkładowi.Więc z nieskończenie małą szansą, że cały eksperyment może po prostu wybuchnąć.To samo z kwantowymi fluktuacjami próżni ...
Z drugiej strony nie widzę praktycznie żadnego problemu, jeśli wszechświat jest teoretycznie nieprzewidywalny.W prawie wszystkich naszych symulacjach i prognozach w świecie rzeczywistym wygładzamy mało prawdopodobne zdarzenia zewnętrzne.Jeśli chcę przewidzieć, gdzie księżyc będzie jutro, nie biorę pod uwagę, że ktoś może go w międzyczasie wysadzić, nawet jeśli jest to możliwe przy prędkości poniżej światła.Byłoby to samo - oczywiście w każdej chwili mogłoby się pojawić coś niezwykle szybkiego i zmienić wszystko, ale wymagałoby to tak dużo energii, aby było tak szybko, że tak naprawdę nigdy by się nie wydarzyło ...
@immibis W rozsądnej skali wszechświat wydaje się jednolity.Tak więc powłoka wszechświata o grubości miliona lat świetlnych i grubości 1 miliarda lat świetlnych, 10 miliardów lat świetlnych, 100 miliardów lat świetlnych, 1000 miliardów lat świetlnych, 10000 miliardów lat świetlnych, 100000 miliardów lat świetlnych - wszystko to składa się na taką samą ilość.Pierwsza z tych powłok generuje pojedynczy foton uderzający w twoje ciało przez całe życie.Cóż, takich muszli jest nieskończona liczba.I coś, co nieskończoność, zrujnuje twój dzień.
Ciekawa perspektywa.Wydaje się to sugerować, że jedynym powodem, dla którego czas nie jest absolutny, jest to, że na początku wszechświata pojawi się dowolna liczba obiektów, które nie są przyczyną niczego innego, podczas gdy tak naprawdę każdy efekt może być wynikiem pojedynczego inicjału"przyczyna".Kiedy staw jest zaśmiecony falami i hałasem, rozsądnie jest założyć, że zaczął się od upadku jednej skały, choć nie zawsze tak było.
co masz na myśli mówiąc „przewidywanie”?nie możemy przewidzieć przyszłości w prawdziwym życiu, więc powoduje to dla mnie zamieszanie
anna v
2016-01-21 16:34:27 UTC
view on stackexchange narkive permalink

Fizyka to dyscyplina naukowa, w której obserwacje i pomiary są wyposażone w modele matematyczne, które opisują istniejące dane i skutecznie przewidują nowe wartości dla nowych warunków brzegowych. Kiedy tak się dzieje, mówi się, że model został zweryfikowany.

Jeśli nowe eksperymenty i obserwacje powinny sfałszować model, trzeba będzie ponownie zbadać założenia, a nawet poszukać nowego modelu.

Obecnie sprawdzonym modelem cząstek elementarnych, który mamy, jest model standardowy, który wykorzystuje relatywistyczną mechanikę kwantową i był wielokrotnie testowany w ramach eksperymentów laboratoryjnych i obserwacyjnych. Ten model matematyczny, ponieważ zawiera szczególną teorię względności, zgadza się z obserwacją, że prędkość światła jest stałą cw próżni. To prawda, że ​​wartość c jest nieoczekiwana w tej dyskusji. Kwestionowane jest istnienie granicy, a jedyną możliwą odpowiedzią jest: ponieważ model teoretyczny zgadza się z eksperymentem i jest bardzo przewidywalny.

Jeśli nowe dane fałszują model standardowy do tego stopnia, że ​​konieczny jest nowy model teoretyczny, ten nowy model będzie musiał uwzględniać istniejącą strukturę dla przypadków, w których został zweryfikowany, w tym prędkość graniczną światła. Model standardowy stałby się ograniczającym przypadkiem dla nowej teorii, dla zweryfikowanych energii i warunków brzegowych, w podobny sposób, w jaki fizyka Newtona wyłania się ze szczególnej teorii względności na granicy niskich energii.

@annav, więc najważniejsze jest to, że najbardziej podstawowym wyjaśnieniem, jakie mamy teraz, jest to, że „po prostu jest”, prawda?
@LandosAdam Tak.Teoria, która wymaga ograniczonej i stałej prędkości dla cząstek o zerowej masie, wywodzi się z danych.Fizyka nie odpowiada ostatecznie na pytania „dlaczego”.Mówi „jak” z przyjętym modelem można dojść do obserwacji.Gdybyśmy zaobserwowali zmienną prędkość światła, opracowalibyśmy inną teorię.
@annav Wiem, że to w pewnym sensie piękno jego obiektywności.Ale czasami teoria może wyjaśnić pewne rzeczy, które wcześniej uważano za fundamentalne.Dlatego zadałem to pytanie.A sposób, w jaki na to patrzę, to fakt, że jest szansa, że można by to wyprowadzić z bardziej fundamentalnych praw, których nie jesteśmy świadomi, a jednocześnie dodaje mi entuzjazmu.Ale kluczowym słowem tutaj jest „MOŻE”, ponieważ równie dobrze mogłoby być tak, jak powiedziałeś.
@LandosAdam: dokładnie.Jeśli zapytamy „dlaczego obiekty poruszają się wokół Słońca / mijają je w przekrojach stożkowych?”, To fizyka może powiedzieć „dlaczego”, czyli w kategoriach innych właściwości modelu, które można w pewnym sensie postrzegać jako bardziej fundamentalne:-przyspieszenie kwadratowe daje pewne cechy geometryczne.Oczywiście nadal nie jest to „ostateczna” odpowiedź na pytanie „dlaczego”, ale jest odpowiedzią.Ale jeśli chodzi o to, dlaczego światło nie pojawia się natychmiast, to Anna stwierdza, że jest tylko jedna możliwa odpowiedź, to znaczy model nie zapewnia żadnych „głębszych” podstaw dla zjawiska.
Warto również pamiętać, że to, czy coś odpowiada na pytanie „dlaczego”, jest nieco subiektywne, ponieważ mówimy o uproszczeniu wyjaśnień („wielokrotności 10 kończą się na 0, ponieważ zapisujemy je na podstawie 10”)nie fizyczne („Boli mnie stopa, bo jest bardziej miękka niż skała, którą właśnie kopnąłem”).Inni doszli więc do wniosku, że * istnieje * odpowiedź na pytanie „dlaczego” i faktycznie mogą mówić o cechach modelu, które * dla nich * częściowo wyjaśniają istnienie w modelu granicy.
@SteveJessop Zgadzam się z tym, co mówisz, chociaż nie rozumiem, dlaczego odpowiedź na pytanie „jaki jest najbardziej podstawowy powód, dla którego stało się to X” jest subiektywna.To nie jest kwestia opinii.Wydaje się to kwestią opinii, ponieważ niektórzy ludzie wiedzą więcej niż inni, dlatego otrzymujemy wiele odpowiedzi.Ale jestem pewien, że jeśli zagłębisz się w większość odpowiedzi, znajdziesz najbardziej fundamentalną odpowiedź na tej stronie, aw tym przypadku nie wiemy, dlaczego się za nią kryje (jeśli istnieje dlaczego).
@LandosAdam: Jeśli nie widzisz, dlaczego odpowiedź na pytanie „co jest najbardziej fundamentalnym powodem X” jest subiektywna, to musisz pomyśleć o *, jaka jest twoja definicja „fundamentalnego” *.Gwarantuję, że nie jest to obiektywne.
@QuanticMan właśnie zobaczyła tę dyskusję.Fizyka używa matematyki.W matematyce powinieneś wiedzieć, że wybór aksjomatów jest subiektywny, zwykle zależy od minimalizacji kroków.Prawdą jest, że w przypadku aksjomatycznej teorii matematycznej można by podstawić twierdzenie za aksjomat, a wtedy aksjomat staje się twierdzeniem, tj. Dającym się udowodnić na podstawie aksjomatów.W fizyce istnieją postulaty, które wybierają podzbiór matematyki aksjomatycznej, który pasuje do danych.W tym samym sensie, jak w przypadku aksjomatów, można by wybrać różne postulaty i użyć matematyki do udowodnienia poprzednich postulatów, subiektywnej części.
@annav, Twoje komentarze i posty są ZAWSZE trafne i bardzo wnikliwe.Publikuj dalej ... NAPRAWDĘ lubię czytać Twoje odpowiedzi.
Miałem tę teorię od dawna.Wierzę, że prędkość światła jest ograniczona do tego, czym jest, ponieważ przepływ CZASU jest dokładnie taki sam (w innych jednostkach) i wszystko, co jest szybsze niż światło, przebiłoby się przez CZAS w przyszłość (lub nigdzie), aby nigdy więcej nie być widzianym.Nawet jeśli przyszłość staje się TERAZ, RZECZ wciąż porusza się do nowej przyszłości nowego TERAZ.Tak więc, gdyby całe światło poruszało się szybciej - nie byłoby żadnego światła do zobaczenia i nie byłoby nas tutaj.
@annav Masz na myśli to, że nawet jeśli osiągniemy najbardziej fundamentalne zrozumienie naszego Wszechświata, jakie możemy, ktoś mógłby odpowiedzieć na to pytanie, zmieniając aksjomat z twierdzeniem i podając inne wyjaśnienie?Dlatego wyjaśnienie „podstawowego” powodu uważa się za subiektywne?(Czy moglibyśmy również przedłużyć tę rozmowę na czacie?)
@annav http: // chat.stackexchange.com / rooms / 58305 / on-the-do-my-know-why-there-is-a-speed-limit-in-our-universe-question
Teoretyczne idee nie istnieją w teoretycznej próżni.Apele do eksperymentów to nie jedyne możliwe odpowiedzi na pytania „dlaczego”.
@BenCrowell Najwyraźniej się nie zgadzamy.„Dlaczego” pytania po przejściu * „jak * matematycznie pasują do modelu teoretycznego” kończą się na postulatach / zasadach / prawach, które są destylacją obserwacji / eksperymentów, dodatkowymi aksjomatami nałożonymi na matematykę w celu wybrania podzbiorurozwiązania istotne dla danych.Wtedy jedyną odpowiedzią jest „ponieważ to właśnie zostało zaobserwowane”.
@annav Po co w ogóle cokolwiek wyjaśniać?Po co w ogóle formułować jakąkolwiek teorię?Tylko po to, abyś mógł łatwo zapamiętać dane?Myślę, że mamy teraz dobre komputery, więc nie musimy o niczym pamiętać.Zatem uważam, że duża tabela wszystkich obserwacji jest najlepszą fizyką, jaką mogliśmy zrobić w waszym wszechświecie.Cieszę się, że nie mieszkam w Twoim wszechświecie.
Fizyka teoretyczna nie jest mnemonikiem służącym do zapamiętywania wyników eksperymentów.W dowolnym momencie najbardziej podstawowa teoria, którą mamy _ jest_ dokładnie odpowiedzią na ** dlaczego ** zachodzą wszystkie zjawiska, które można wyjaśnić za pomocą tej podstawowej teorii.Byłoby głupie lub nieuczciwe stwierdzenie: „nie możemy wyjaśnić, dlaczego mamy prawo odwrotnej siły kwadratowej dla grawitacji lub elektromagnetyzmu, poza stwierdzeniem, że tak się stało”.Podstawowe zasady teorii GR i YM _ są_ powodem, dla którego mamy te prawa odwrotnych kwadratów.
@Dvij Nie zgadzam się.Najpierw zbadaliśmy i znaleźliśmy zachowanie 1 / r ^ 2, a następnie użyliśmy go do PRZEWIDYWANIA przyszłego zachowania.Banki danych, podobnie jak mapy, nie mają zdolności predykcyjnej.W fizyce ważna jest możliwość prognozowania z wykorzystaniem narzędzi matematycznych (modeli matematycznych).Prawidłowe prognozy weryfikują narzędzie, nieprawidłowe prowadzą do zmian w modelach.Teorie fizyczne są formułowane w taki sposób, aby można było przewidzieć przyszłe konfiguracje i sprawdzić teorię z nowymi danymi.
Solomon Slow
2016-01-21 20:50:14 UTC
view on stackexchange narkive permalink

Maxwell przeprowadził rygorystyczne, matematyczne badanie właściwości elektryczności i magnetyzmu i udowodnił, że musi istnieć zjawisko, które nazwał falami elektromagnetycznymi . Zgodnie z teorią Maxwella, fala elektromagnetyczna musi rozchodzić się ze stałą prędkością, którą nazwał $ c $ i którą można obliczyć na podstawie innych stałych fizycznych, które były znane i mierzalne w tamtym czasie.

Mniej więcej w tym czasie, naukowcy aktywnie debatowali nad naturą światła. Po tym, jak Hertz wykazał istnienie fal elektromagnetycznych w eksperymencie laboratoryjnym, teoria Maxwella nagle stała się najbardziej popularnym wyjaśnieniem dla światła.

Zabawne w teorii Maxwella było to, że prędkość była zależna od tego, kto ją zmierzył. Jeśli ty i ja zmierzyliśmy prędkość fal emanujących z tego samego źródła, oboje powinniśmy otrzymać ten sam wynik, niezależnie od naszego ruchu względem źródła lub siebie nawzajem.

Niektórzy fizycy odkryli, że być niepokojące i próbowali to usprawiedliwić na różne sposoby. Wielkim osiągnięciem Einsteina było połączenie różnych pomysłów w jedną spójną, rygorystyczną teorię matematyczną. Jedną z konsekwencji jego teorii - udowodnioną matematycznie - jest to, że jeśli cokolwiek ma charakterystyczną prędkość, która musi być taka sama dla każdego obserwatora, to nikt nigdy nie może zaobserwować niczego poruszającego się szybciej niż ta prędkość.

Tak więc, uniwersalne ograniczenie prędkości jest matematyczną konsekwencją pewnych mierzalnych i, o ile ktoś wie, podstawowych stałych fizycznych; np. przepuszczalność wolnej przestrzeni.

Twoje pytanie sprowadza się do tego, „dlaczego wszechświat ma te właściwości?”

Za każdym razem, gdy fizycy odpowiadają pytanie „dlaczego”, odpowiedź zawsze spoczywa na głębszych poziomach „dlaczego?”

W rzeczywistości równania pokazują prędkość jako funkcję stałych magnetycznych i elektrycznych.To dalsza obserwacja, że nie zachowują się one jak normalne menium, względem którego obserwator się porusza, ale uparcie odczytują tę samą wartość niezależnie od ruchu.
_ "Zabawne w teorii Maxwella było to, że ** prędkość była względna ** w stosunku do tego, kto ją zmierzył."ramka odniesienia?
* Zabawną rzeczą w teorii Maxwella było to, że prędkość była zależna od tego, kto ją zmierzył. * - Nie sądzę, aby to prawda, ponieważ gdyby tak było, wynik eksperymentu Michelsona-Morleya nie byłby takizaszokować.Czy coś mi brakuje?
** prędkość była względna **: Oznacza, że obliczona prędkość c jest taka sama dla dowolnego obserwatora.Jest to ta sama dwuznaczność zdań, która nadała * względności * jego nazwę, ponieważ Einstein zauważa ten sam problem z nim po angielsku.Prędkość fali c względem Alicji, a prędkość c względem Boba.
Wskazuje to dokładnie we właściwym kierunku i zasługuje na znacznie większe poparcie niż obecna „najwyższa” odpowiedź.Byłbym bardzo zadowolony, gdybyś rozszerzył to, aby uwzględnić podstawowe punkty wyprowadzeń (zarówno Maxwella, jak i Einsteina).
@Floris, Jestem tu tylko turystą.Chciałbym kiedyś nauczyć się fizyki.Spędziłem więcej czasu, co lubię się przyznać, pracując nad „Drogą do rzeczywistości” Rogera Penrose'a.Moim największym wnioskiem było to, że fizyka to duży temat.Ta książka to w zasadzie tysiąc sto stron krótkiego wprowadzenia do niektórych matematycznych technik i form, których należałoby się nauczyć, aby zrozumieć fizykę.Może kiedy (jeśli!) Przejdę na emeryturę ...
user46925
2016-01-21 04:38:55 UTC
view on stackexchange narkive permalink

W fizyce nie można zapytać / odpowiedzieć dlaczego bez niejasności. Teraz widzimy, że prędkość światła jest skończona i wydaje się być największą prędkością dla energii.

Skuteczne teorie zostały zbudowane wokół tego ograniczenia i są one spójne, ponieważ zależą od urządzeń pomiarowych, które są oparte na technologii / naukach, które wszystkie mają wbudowane c. We współczesnych naukach nie obchodzi nas, co się dzieje, ale to, co mierzą te urządzenia.

Walidacja tych teorii pozwala łatwo powiedzieć, że istnieje uniwersalna prędkość maksymalna. W rzeczywistości istnieje maksymalna prędkość poruszającego się obiektu wytworzonego energią, gdy jest mierzona w przestrzeni statycznej. Nie jest to dokładne w rozszerzającym się wszechświecie lub w innych krytycznych kontekstach relatywistycznych. Nie jest dokładne, jeśli nie jest to energia, tj. O przypuszczalnym wpływie, który zostałby wymieniony przez splątane cząstki. Mimo to nie można podróżować szybciej niż światło, chyba że we śnie.

Współczesna fizyka jest nowa. Rozciąga się we wszystkich kierunkach. Nadal trudno jest dokonać syntezy wszystkiego, co wiadomo, co jest przydatne, a co nie jest spójne i istotne. Być może skończona wartość c (i nie tylko c) będzie pochodzić z analizy kwantowych pól głębokich, w taki sam sposób, w jaki teoria względności była zgodna z bardzo bogatą teorią Maxwella. Tymczasem społeczność naukowa nie jest świadoma takiej nowej analizy, nawet jeśli została już opublikowana w jakimś mało znanym repozytorium.

To brzmi: „Dlaczego wiemy, że w naszym wszechświecie obowiązuje ograniczenie prędkości?”co jest zupełnie innym pytaniem niż „Czy wiemy, dlaczego w naszym wszechświecie obowiązuje ograniczenie prędkości?”
@immibis Próbowałem pokazać, że nie ma odpowiedzi na to * dlaczego *, ponieważ jest to fakt eksperymentalny, który jest traktowany jako postulat, nawet jeśli można dyskutować o koncepcji prędkości.Być może jego status będzie ewoluował wraz z większą wiedzą i większą syntezą, ale dziś kto wie, jak wydedukować ograniczenie prędkości z innych założeń?
W naukach ścisłych jest problem z pytaniem „dlaczego”.W codziennym języku angielskim „dlaczego” i „jak” są często synonimami: odpowiedź na pytanie „Dlaczego niebo jest niebieskie” jest taka sama, jak „Jak jest błękitne niebo”.Ale w przypadku głębszych pytań „dlaczego” staje się bez znaczenia, ponieważ jest to kwestia teleologiczna (a nie _teo_logiczna) - ostatecznie chodzi o pytanie o _cel_ lub _końcowy cel_.Ale prawa natury nie są stworzone do określonego celu;po prostu są.Zatem „jak” jest naprawdę jedynym pytaniem, na które nauka może odpowiedzieć.W tym przypadku sposób wynika z równań.
@user151841 tak, ponieważ na * Why * należy odpowiedzieć logiczną demonstracją w aktualnej teorii.Ale nie możemy przedstawić postulatu.Ten pozostaje na solidnych obserwacjach i doświadczonych teoriach
@user151841 Chociaż częściowo się z tobą zgadzam i mam tendencję do umieszczania tych pytań w jakiejś części metafizyki, nie zgadzam się z tobą, redukując je do argumentu teleologicznego.To nie to samo, co pytanie, dlaczego wartość $ \ pi $ jest taka, jaka jest, co byłoby bez znaczenia.Może się tu raczej zdarzyć, że alternatywa nie jest nawet spójna w sensie matematycznym, co dałoby odpowiedź nieopartą na argumentach teleologicznych.
@G.Bergeron Nie zgadzam się;dla wyrafinowanych odbiorców wyjaśnienia, które są epifenominalnymi prostszymi zjawiskami lub są wyłaniającymi się zachowaniami lub właściwościami, są zwykle satysfakcjonujące w przypadku pytania „dlaczego”.Podejrzewam, że właśnie o to pytają
Jeśli fizyka nie da nam odpowiedzi na pytanie „dlaczego” powinniśmy zamiast tego czytać książki religijne, aby poznać odpowiedź?
Mikael Fremling
2016-01-25 17:50:30 UTC
view on stackexchange narkive permalink

To pytanie zaowocowało kilkoma interesującymi odpowiedziami, do których również chciałbym wnieść swój wkład. Powinno być całkowicie jasne, że żyjemy w świecie o skończonej górnej prędkości, a wiele odpowiedzi dotyczyło konsekwencji i przyczyn takiego stanu rzeczy.

Chciałbym jednak zwrócić uwagę na pewien aspekt Wydaje się, że w innych odpowiedziach całkowicie zapomniano. Gdyby prędkość światła była nieskończona, w ogóle nie mielibyśmy światła.

Aby to zobaczyć, spójrz ponownie na równania Maxwella. Zauważ, że w nich $ c = \ frac {1} {\ sqrt {\ mu_0 \ epsilon_0}} $, więc jeśli ustawisz $ c \ na \ infty $, to albo (lub oba) z $ \ mu_0 $ i $ \ epsilon_0 $ musiałby wynosić zero. To skutecznie zabije istnienie dynamicznych pól magnetycznych.

Szczególnie w przypadku światła oznacza to, że $ \ nabla \ times B = \ frac1 {c ^ 2} \ frac {\ części E} {\ części t} \ do 0 $, więc pola magnetyczne byłyby statyczne (i o zerowej intensywności, pamiętajcie o braku monopoli magnetycznych). Zatem jedyną rzeczą, która pozostała z elektromagnetyzmu byłaby po prostu elektrostatyka.

Fizycznie to również ma sens, jeśli $ c \ do \ infty $, to odpowiedź pola elektrycznego na jakiekolwiek przegrupowanie ładunków byłaby natychmiastowa, więc nie ma miejsca (czasu?) na odpowiedź pola magnetycznego.

Również myśląc o polach magnetycznych (a zwłaszcza siła Lorentza), sensowne jest również zanikanie pól magnetycznych. Jeśli $ c \ do \ infty $ nie ma skrócenia długości, a więc nie będzie sił Lorentza na żadnych cząstkach.

Zatem kiedy mówimy o sygnałach propagujących się nieskończenie szybko, wątpliwe jest, do jakich sygnałów się odnosimy do.

Myślę, że twoja odpowiedź jest pierwszą, która naprawdę pokazuje, co się psuje, gdy $ c $ dąży do nieskończoności, bez faktycznego przywoływania okrągłego argumentu „przyczynowość ponieważ SR i nieskończone c brak przyczynowości” itd. Dziękuję i +1
Szczerze mówiąc, tak naprawdę nie przez argument, że $ c \ do \ infty $ oznacza brak związku przyczynowego.To tylko granica Newtona, gdzie czas jest absolutny, i o ile wiem, to działa dobrze.
Nieskończoność jest znacznie większa niż prędkość światła.Dlaczego prędkość światła jest taka sama wszędzie i przez cały czas?Ponieważ przestrzeń zmienia gęstość, co jest obserwowane przez przesunięcie ku czerwieni i tym podobne, ale czy czas (prędkość światła) również się nie zmienia?
@CeesTimmerman: Powiedziałbym, że nie jesteśmy pewni, dlaczego $ c $ ma taką samą wartość w całej czasoprzestrzeni.Najsilniejszym argumentem byłaby ogólna zasada względności.Jestem jednak pewien, że gdyby prędkość światła była zależna od pozycji (czasoprzestrzeni), to można było uzyskać dziwne efekty, które, miejmy nadzieję, byłyby wykrywalne z Ziemi.
Powiedzenie „prędkość światła nie może być nieskończona” wcale nie jest tym samym, co powiedzenie „nic nie może iść szybciej”.
@Floris: Zgadzam się, ta ostatnia jest konsekwencją szczególnej (lub ogólnej) teorii względności.A więc wierzymy w to, że nic nie może iść szybciej niż prędkość światła, nawet neutrina;).Pierwsze stwierdzenie jest bardziej faktem eksperymentalnym.Prędkość światła jest skończona.Nie wyklucza to jednak istnienia spójnych teorii na granicy, na której od $ c \ do \ infty $.Ten post pokazał tylko jeden (z wielu) przykładów, dlaczego ten limit jest o wiele bardziej nudny niż w przypadku skończonego $ c $.
Arnold Neumaier
2016-02-09 16:03:15 UTC
view on stackexchange narkive permalink

Dlaczego mamy uniwersalne ograniczenie prędkości? Czy istnieje bardziej fundamentalne prawo, które mówi nam, dlaczego tak jest?

Bardziej fundamentalne prawa to przyczynowość i lokalność. Przyczynowość wyraża fakt (lub założenie), że skutki nie mogą poprzedzać przyczyn, a lokalność wyraża fakt (lub założenie), że fundamentalne relacje przyczynowe są opisane równaniami różniczkowymi.

Biorąc pod uwagę te dwie podstawowe zasady, logika matematyczna mówi, że równania różniczkowe są albo paraboliczne (podobne do równania ciepła), albo symetryczne hiperboliczne (podobne do równania falowego).

Jeśli są paraboliczne, nie ma ograniczenia prędkości. Na przykład, zgodnie z równaniem ciepła, ciepło rozchodzi się natychmiastowo do dowolnie odległych miejsc, choć jest tłumione wykładniczo wraz z odległością.

Jeśli są one symetryczne, teoria matematyczna zakłada skończoną prędkość propagacji. Na przykład tak jest w przypadku równań Maxwella, które ograniczają prędkość sygnałów elektromagnetycznych do liczby zwanej prędkością światła.

Jest faktem doświadczalnym, że Natura zachowuje się zgodnie z drugą możliwością - nawet niezależną rozważań nad prędkością światła. Istnieją przytłaczające dowody na to, że wszystkie podstawowe procesy w Naturze są symetryczne i hiperboliczne. Równomierne ciepło - równanie ciepła to tylko najprostsze przybliżenie, w którym traci się ograniczenie prędkości. Ale bardziej wyrafinowane wyprowadzenia z nierównowagowej mechaniki statystycznej dają symetryczne równania hiperboliczne, które stają się paraboliczne dopiero po dalszym przybliżeniu.

Jest bardzo prawdopodobne, że graniczną prędkością jest prędkość światła, ale niekoniecznie. Jest to związane z założeniem, że fotony są bezmasowe. Gdyby fotony były masywne, ale grawitony nie miałyby masy, prędkość światła byłaby mniejsza niż teoretyczna granica prędkości sygnałów we Wszechświecie - czyli prędkość grawitacji.

Jednak zgodnie z przeglądem cząstek przeprowadzonym przez Particle Data Group, górne granice masy fotonu są niezwykle małe, a obserwacje są obecnie w pełni zgodne z założeniem o bezmasowych fotonach.

Dziękuję Ci!Szukałem tej odpowiedzi.Chciałem zapytać, czy przyczynowość i lokalność (hiperboliczność) różnią się w jakikolwiek sposób.Jeśli hiperboliczność zapewnia górną granicę prędkości, powiedzmy prędkość światła, oznacza to również zakaz ponadświetlnego transferu informacji, a zatem pociąga za sobą przyczynowość.Więc to to samo, prawda?
@NanashiNoGombe: Lokalność oznacza opisywalną za pomocą równania różniczkowego.Równanie paraboliczne jest lokalne, ale nie przyczynowe.
Dziękuję za szybką odpowiedź.Pytałem, czy hiperboliczność to to samo, co relatywistyczna przyczynowość?Myślę, że to jest.Proszę, popraw mnie jeśli się mylę.
@NanashiNoGombe: Tak samo jest w klasycznej fizyce relatywistycznej.W fizyce kwantowej relacja jest bardziej skomplikowana.
noncom
2016-01-21 17:45:37 UTC
view on stackexchange narkive permalink

Istnienie ograniczenia prędkości jest związane z istnieniem czasu [AKTUALIZACJA: czas jest miarą dostępną tylko wtedy, gdy $ c $ jest ograniczone. Jeśli nie zgadzasz się, podaj sposób pomiaru czasu, w którym $ c $ jest nieskończony, przed głosowaniem w dół]. Gdyby nie było ograniczenia prędkości, wszystko wydarzyłoby się natychmiast. Ponadto żadne fale w jakiejkolwiek materii nie zostałyby dotknięte i rozprzestrzeniłyby się chwilowo. Czas zniknąłby (podobnie jak odległość, a co za tym idzie, przestrzeń, tak przy okazji).

To to samo, co „dlaczego jest czas?”. Natychmiastowy transfer energii, który jest obecnie ograniczony, zmieniłby świat, jaki znamy i nie byłby tym światem, który znamy. Fizyka Newtona zniknęłaby jako koncepcja, ponieważ sama materia już by tak nie działała. Jak również pojęcie formy. Konsekwencje dotknęłyby wszystkiego. Jednak my tego nie obserwujemy, przestrzegamy granicy.

W materii Uniwersalnej istnieje pewna nieodłączna separacja, która pozwala jej istnieć w taki sposób, w jaki ją znamy / postrzegamy. Jeśli istnieje świat bez ograniczeń, nie wyłoniliśmy się w nim, pojawiliśmy się tutaj.

Niezupełnie odpowiedź, ale nie ma nic więcej do powiedzenia

AKTUALIZACJA

W odpowiedzi na komentarz @Davors:

Trudno sobie wyobrazić, co by się dokładnie wydarzyło, ponieważ nie możemy być pewni, jaka jest rzeczywista podstawowa struktura rzeczywistości, która nadrabia prędkość światła i jak jest spleciona z resztą rzeczy. To znaczy - w jaki sposób pozostałe 3 siły utworzyłyby materię, gdyby EM była natychmiastowa. Ale przyjrzyjmy się kilku opcjom, które wspierają tę koncepcję:

  1. Są 4 siły i jeśli siła EM natychmiast przeniosłaby całą energię, to nawet gdyby 3 inne siły nadal utrzymywały, unieważniłoby większość struktur większych niż atomy materii, którą obecnie widzimy. Ponieważ na poziomie makro tylko grawitacja i EM mają znaczenie, a grawitacja nie będzie miała większego sensu w tym scenariuszu, wtedy wszystkie procesy, które są prowadzone przez siły EM, będą natychmiastowe.

  2. Żadna makrostruktura nie mogłaby istnieć, a przy nieskończenie szybkim przebiegu wszystkich procesów EM nie byłoby możliwości dowiedzenia się czegokolwiek o stanach elektronów w atomach. Staną się nieskończenie wszystkimi możliwymi stanami. Cała możliwa absorpcja i emisja nastąpi jednocześnie. Nie jestem pewien, nawet czy atomy wytrzymają.

  3. Spróbuj wstawić nieskończoność zamiast $ c $ we wszystkich relacjach i zobacz, co się stanie. Ponadto, ponieważ wszystkie prędkości można skutecznie zmierzyć jako ułamek $ c $, to jeśli $ c = \ infty $, wszystkie inne prędkości również będą nieskończone niezależnie od współczynnika ułamka.

  4. Zobacz odpowiedź @Nikos M.

Komentarze nie służą do rozszerzonej dyskusji;ta rozmowa została [przeniesiona do czatu] (http://chat.stackexchange.com/rooms/34701/discussion-on-answer-by-noncom-do-we-know-why-there-is-a-speed-limit-in-our-univ).
Mówisz, że gdyby przyczynowość nie miała ograniczenia prędkości, to wszystkie procesy przebiegałyby z nieskończoną prędkością?
Michael
2016-01-30 03:53:29 UTC
view on stackexchange narkive permalink

$ \ hspace {50px} $
Powyższe zdjęcie narysowałem, aby rozwinąć wspaniałą odpowiedź Kostyi.

Wyobraź sobie ludzi, którzy mierzą wzrost budynków w stopniach kąta widoczności budynków z określonej stałej odległości. Nie jest to wcale nieracjonalne, jeśli odległość C jest wystarczająco duża w porównaniu do wysokości budynku ”. Jednak w przypadku wyższych budynków można zauważyć, że ich wysokość kątowa nie jest addytywna. Również maksymalna możliwa wysokość kątowa jest ustalona na bezwzględną wartość 90 stopni.

Jest to bardzo podobne do sposobu, w jaki ludzie mierzą prędkość: wybraliśmy pewną miarę „odległość / czas”, która ma sens przy mniejszych prędkościach, ale przy wyższych prędkościach nie jest addytywny. Jest też nieosiągalna „maksymalna” prędkość, prędkość światła.

Jednak powyższy problem wynika wyłącznie z niewłaściwego wyboru pomiaru prędkości. Jak wyjaśnił Kostya, „właściwym” wyborem pomiaru prędkości jest „szybkość”. A szybkość jest addytywna i nieograniczona.

Ziezi
2016-01-21 23:11:56 UTC
view on stackexchange narkive permalink

Czy wiemy, DLACZEGO w naszym wszechświecie obowiązuje ograniczenie prędkości?

Twoje pytanie jest podobne do:

„Czy wiemy DLACZEGO istnieje ograniczenie długości? ”

W ten sam sposób potrzebujemy skończonych długości do pomiaru rozmiaru lub odstępu między dwoma punktami w 3D - przestrzeni euklidesowej , potrzebujemy skończona prędkość światła do pomiaru odstępu między zdarzeniami w 4D - przestrzeni Minkowskiego . Minkowski rozwinął swoją teorię, aby rozszerzyć równanie Maxwella w czterech wymiarach. Aby $ s ^ 2 $ w przestrzeni Minkowskiego, (2), zachował niezmienność, jako rozszerzenie twierdzenia Pitagorasa, (1), które w trzy wymiary to: $$ s ^ 2 = x ^ 2 + y ^ 2 + z ^ 2 \ tag {1} \ ,, $$

iw czterech wymiarach staje się: $$ s ^ 2 = x ^ 2 + y ^ 2 + z ^ 2 - {\ left (ct \ right)} ^ 2 \ tag { 2} \ ,, $$ $ c $ musi być nie tylko skończona, ale taka sama dla wszystkich ramek odniesienia, co potwierdzają obserwowane dowody, że światło Prędkość (fal EM) była niezależna od układu odniesienia obserwacji.

Prędkość światła musi mieć granicę, tj. Być skończona, aby Szczególna Teoria Względności działała:

Jeśli wróć do słynnych eksperymentów myślowych A. Einsteina, a konkretnie tego, w którym jest dwóch obserwatorów, jeden nieruchomy, $ A $ , stojący na stacji kolejowej, a drugi, $ B $ , poruszając się stojąc w pociąg, który przejeżdża przez stację kolejową.

enter image description here

Teraz, gdy pociąg przejeżdża i $ A $ i $ B $ są dokładnie naprzeciw siebie, piorun uderza po obu stronach $ A $ , w tej samej odległości . $ A $ widzi je jednocześnie:

enter image description here

Ponieważ jednak $ B $ przesuwa się względem nich, tj. od jednego do drugiego, widzi je kolejno:

enter image description here

...right?

Cóż, NIE, spowodowałoby to różny pomiar światła w różnych ramach referencyjnych, coś, co została obalona przez eksperyment Michelsona-Morleya , w którym Ziemia była pociągiem :
$ \ hspace {100px} $ ,

i zmierzona prędkość światła w dwóch prostopadłych kierunkach:

enter image description here

zakładając, że światło poruszające się w kierunku zgodnym z kierunkiem ruchu Ziemi musiałoby być mniejsze (podobnie jak osoba $ B $ , która była na pociąg) niż inne, założenie, które zostało naukowo udowodnione.

W konsekwencji prędkość światła jest stała i każdy, kto ją mierzy, znajdzie tę samą wartość, niezależnie od jego prędkości lub innymi słowy prędkość światła jest niezmienna . Niezmienność można porównać z teorią względności, na przykład względnością czasu, która, nawiasem mówiąc, została użyta do opisu dwóch osób, $ A $ i $ B $ , obserwuj tę samą prędkość światła, a mianowicie, ponieważ czas poruszającej się osoby $ B $ tyka wolniej, w Ogólnie rzecz biorąc, im większa prędkość względna między dwoma obserwatorami, tym większa różnica w tempie tykania ich zegarków, tj. dylatacja czasu .

Wreszcie dylatację czasu można zaobserwować w obecności obiektu o masie, który generuje pole grawitacyjne, lub w kategoriach ogólnej teorii względności w rozciągniętej czasoprzestrzeni, co spowoduje czas obserwatora znajdującego się bliżej obiektu masowego, aby tykał w wolniejszym tempie, tj. dylatacja czasu i odpowiednio, obserwator znajdujący się w większej odległości będzie obserwował tykanie swojego zegarka z większą szybkością.

Jak widzisz, czas jest względny, przestrzeń się rozciąga, a prędkość światła jest stałą, o skończonej wartości, która „trzyma je razem” i „synchronizuje” , definiowanie jednoczesności zdarzeń. Ponadto za jego pomocą możemy zdefiniować niezmienny odstęp między dwoma punktami w czasoprzestrzeni, czyli między dwoma zdarzeniami . Przedziały czasoprzestrzenne zależą od czasowego i przestrzennego oddzielenia dwóch punktów i mogą być: czasowe , lekkie (odległość czasowa = odległość przestrzenna ) lub podobna do przestrzeni (odległość czasowa < odległość przestrzenna) . Dzięki temu Szczególna Teoria Względności jest udaną teorią, z długą listą dowodów potwierdzających eksperyment.

Edycja:

W odpowiedzi na pierwszy komentarz, który dowodzi, że dylatacja czasu jest wynikiem stałości prędkości światła, przedstawię wam przykład odwrotny, tj. stałość prędkości światła może być pokazana jako bezpośrednia konsekwencja dylatacji prędkości:

Rozważmy hipotetyczny zegar zwany zegarem fotonowym. W nim światło odbija się tam iz powrotem między lusterkami, a za każdym razem, gdy światło pada na dane lustro, zegar tyka raz. Jeśli ten zegar jest w ruchu bezwładnościowym względem obserwatora, to dylatacja prędkości w czasie spowoduje, że tykanie będzie wolniejsze, podobnie jak wszystkie inne rodzaje zegarów. Jednakże, ponieważ zegar się porusza, impuls świetlny wytyczy dłuższą, nachyloną ścieżkę między lusterkami. Wynik netto dylatacji czasu prędkości i wzrostu długości ścieżki jest taki, że prędkość światła w poruszającym się zegarze fotonowym pozostaje równa prędkości światła w pozostałym zegarze fotonowym. Innymi słowy, prędkość światła pozostaje stała.

Ponadto transformacja Lorentza (LT), którą wyprowadził Joseph Larmor [1] w 1897 i Lorentz (1899, 1904) [2], bezpośrednio przewidywał dylatację czasu. Faktycznie, dylatacja czasu przez czynnik Lorentza została poprawnie przewidziana przez Josepha Larmora (1897) [3] na długo przed opublikowaniem przez Einsteina swojej pracy w 1905 roku.


Twoje pytanie wydaje się nieco Filozoficzne, z tego, co wiemy, wartość $ c $ może być powiązana z samą własnością czasoprzestrzeni, określoną wraz z innymi Fundamentalnymi stałymi fizycznymi podczas Wielkiego Wybuchu, co jest trudne do zaobserwowania i uświadomienia sobie, podobnie do tego, dlaczego ryby nie są świadome całej wody wokół niej.

[1] Larmor, J. (1897), „O dynamicznej teorii ośrodka elektrycznego i świecącego”, Philosophical Transactions of the Royal Society190: 205-300.
[2] Lorentz, Hendrik Antoon (1899), „Uproszczona teoria zjawisk elektrycznych i optycznych w ruchomych systemach ”, Proc. Acad. Science Amsterdam I: 427-443; oraz Lorentz, Hendrik Antoon (1904), „Zjawiska elektromagnetyczne w układzie poruszającym się z dowolną prędkością mniejszą niż prędkość światła”, Proc. Natl. Acad. Science Amsterdam IV: 669–678.
[3] Larmor, J. (1897), „On a Dynamical Theory of the Electric and Luminiferous Medium, Part 3, Relations with material media”, Phil. Trans.Roy. Soc. 190: 205-300

MichaelS
2016-01-22 11:00:58 UTC
view on stackexchange narkive permalink

Myślę, że wiele odpowiedzi koncentruje się na niewłaściwej połowie problemu. Mówią ci, skąd wiemy, że istnieją ograniczenia, zamiast wyjaśniać, dlaczego tak musi być.

W większości przypadków nic nie stoi na przeszkodzie stworzeniu wszechświata o nieskończonej prędkości światła, który poza tym jest podobny do naszego 1 . Jest jednak jedna ważna właściwość, którą taki wszechświat musiałby mieć: musi być skończony i / lub niejednorodny.

Jest to związane z Paradoksem Olbera. Zasadniczo wygląda to tak:

  1. Światło gwiazdy w określonej odległości jest odwrotnie proporcjonalne do kwadratu odległości. $ L \ propto {1 \ over D ^ 2} $ .
  2. Liczba gwiazd w danej odległości jest wprost proporcjonalna do kwadratu dystans. $ N \ propto D ^ 2 $ .
  3. Całkowite światło gwiazd w danej odległości jest równe ilości światła na gwiazdę pomnożonej przez liczbę gwiazdy. $ T \ text {light} $ $ = LN \ text {light} $ $ = L {\ text {light} \ over \ text {star}} N \ text {stars} $ . („światło” i „gwiazda (-y)” to jednostki.)
  4. Zatem całkowite światło z jednej odległości jest takie samo jak światło z każdej innej odległości. $ T \ propto LN $ $ \ propto D ^ 2 {1 \ over D ^ 2} $ $ \ propto 1 $ .
  5. Jeśli gwiazdy są równomiernie rozmieszczone we wszechświecie, a wszechświat jest nieskończony, możemy podzielić gwiazdy na nieskończoną liczbę muszli, z których każda ma skończoną, stałą jasność. Suma tego światła jest nieskończona. $ \ sum_ {D = 0} ^ {\ infty} T = \ infty $ .

W prawdziwym wszechświecie nie stanowi to problemu z powodu dwóch efektów: po pierwsze, ekspansja wszechświata oznacza, że ​​każda gwiazda we wszechświecie oddala się od nas (średnio), a tempo recesji jest wprost proporcjonalne do odległość gwiazdy; a po drugie, ponieważ skończona prędkość światła oznacza, że ​​światło z odległych gwiazd potrzebuje więcej czasu, aby dotrzeć do nas z powodu wspomnianej ekspansji.

W połączeniu oznacza to, że światło dociera do nas w ciągu sekundy z dowolnej powłoki gwiazd spada liniowo wraz z odległością. $ L \ propto {1 \ over D} $ . Oznacza to również, że istnieje skończona odległość, w której wszystkie obiekty w tej odległości oddalają się od nas szybciej niż prędkość światła, więc nigdy nie zobaczymy światła spoza tej odległości (skutecznie sprawiając, że wszechświat jest skończony). $ \ sum_ {D = 0} ^ {N, N< \ infty} {T \ over D} < \ infty $ .

Ale jeśli światło podróżowało z nieskończoną prędkością, ekspansja nie pomogła. To oznaczałoby , że jasność Wszechświata powoli spada (gwiazdy są bardziej rozproszone, więc jasność na powłokę jest niższa), ale zmniejszenie nieskończonej jasności niewiele pomaga. Zatem Wszechświat musiałby mieć skończone rozmiary i / lub gwiazdy musiałyby być mniej gęste, im dalej od centrum się znajdujesz.

Alternatywnie możemy założyć, że we wszechświecie światło słabnie szybciej. $ L \ propto {1 \ over D ^ 3} $ czy coś. Ale to już nie ma sensu geometrycznego i wymaga dodatkowego manipulowania przy pracy. Być może światło jest w jakiś sposób absorbowane przez mechanizm ekspansji, przy czym światło jest absorbowane łatwiej, ponieważ jego moc jest bliska zeru. Nieskończona przestrzeń oznacza, że ​​proporcjonalnie nieskończona absorpcja światła skutkuje skończoną szybkością rozszerzania i zwiększa tłumienie światła. Ale to rodzaj bzdur, które wymyśliłem i należę do budowania świata bardziej niż obecna fizyka.

Nawet jeśli Wszechświat byłby skończony, zauważ, że jasność wszystkich gwiazd w naszej galaktyce znacznie przyćmiewa Słońce. Więc naprawdę nie możemy mieć wszechświata prawie identycznego z naszym, jeśli światło nie ma skończonej prędkości.

Ponadto, zauważ, że nie wiem nic o mechanice kwantowej ani o tym, jak prędkość światła wpływa na tę dziedzinę nauki. . Możliwe, że obecna mechanika kwantowa nie mogłaby istnieć z nieskończoną prędkością światła, co można by zadeklarować, oznacza, że ​​każdy wszechświat o nieskończonej prędkości światła jest zupełnie inny od naszego. 1 Jednakże przez „podobny do naszego wszechświata” uważam każdy wszechświat z fizyką cząstek, który pozwala na tworzenie planet, gwiazd, ścieżek neuronowych itp. na poziomie makro, który typowy człowiek rozpoznałby jako podobny. Nie obchodzi mnie, czy „złoto” ma więcej protonów niż „wodór” itp., Znacznie mniej efektów kwantowych.

Sprawdź fizykę serii * ortogonalnej * Grega Egana.Twoje argumenty zakładają stały globalny czas, taki jak Galileo, który * nie * jest jedynym rozwiązaniem.Egan ma 4 wymiary, przy czym czas jest kierunkiem względnym: nieskończone prędkości (dla ramy odbioru ortogonalnych obserwatorów) i brak katastrofy Olbera.
@JDługosz nie paradoksu Olbersa, ale poza tym trochę dziwnych rzeczy!To świetny tekst - zwykle nie jestem fanem SciFi, ale Orthogonal zrobił to za mnie.
kpv
2016-02-10 12:50:05 UTC
view on stackexchange narkive permalink

Na podstawie Twojego pytania wyczuwam, że szukasz prostego i podstawowego wyjaśnienia bez żargonu. Podam uczciwe ujęcie i utrzymam, że jest naprawdę prosty i klasyczny. Jestem myślicielem klasycznym, więc nie mam nawet bardziej złożonego wyjaśnienia. Mam nadzieję, że bardziej wykwalifikowani i akredytowani użytkownicy nie będą krzywo patrzeć na odpowiedź.

Podzielę pytanie na dwie części -

(1) Dlaczego fale elektromagnetyczne mają określoną prędkość (która tak się składa, że ​​c)

Prędkość każdej fali jest własnością ośrodka, przez który się przemieszcza. Zatem pustą przestrzenią jest to, że fale elektromagnetyczne przemieszczają się z określoną prędkością (nie więcej, nie mniej). To właściwość, a nie ograniczenie. Gdyby to była granica, to światło (lub EM) mogłoby podróżować w < c przez pustą przestrzeń. Ale porusza się dokładnie w c, w pustej przestrzeni. Jest to więc właściwość, jeśli masz problem z tym wyjaśnieniem, to powinieneś mieć również problem z - dźwiękiem mającym określoną prędkość w powietrzu i trzeba przejść na bardziej podstawowy poziom. Jeśli akceptujesz określoną prędkość dźwięku, ja będzie oczekiwać, że zaakceptujesz tę właściwość również w przypadku fal elektromagnetycznych. Wartość tej właściwości wynosi ok. James Large również to wskazał w swojej odpowiedzi na twoje pytanie 21 stycznia.

(2) Dlaczego żadne materialne ciało nie może poruszać się szybciej niż c

Jest to bezpośrednia konsekwencja (1) i tak okazuje się ograniczeniem. (Jest to ograniczenie, ponieważ ciała mogą poruszać się z dowolną prędkością, o ile nie przekracza ona c)

Zastanówmy się, jak zwiększyć prędkość masy - przykładamy do niej siłę. Na przykład możemy sprawić, że stojący samochód poruszy się, popychając go rękami. Elektrony w naszych dłoniach i te w samochodzie (tam, gdzie ich dotykamy) odpychają się nawzajem i ta siła odpychania powoduje wzrost prędkości samochodu. Załóżmy, że jechałeś z maksymalną prędkością i samochód przejeżdżał obok Ciebie z prędkością 300 mil / godzinę. Czy możesz zwiększyć prędkość samochodu, popychając go rękoma, gdy mija Cię? Odpowiedź brzmi: nie (normalna istota ludzka nie może poruszać rękami szybciej niż 300 mil / h) .Aby zwiększyć prędkość poruszającego się ciała, siła musi działać na nie szybciej niż poruszające się ciało.

Jakakolwiek siła, którą przyłożymy do przyspieszenia ciała, jest ostatecznie wywierana na ciało jako jedna z podstawowych sił. Wszystkie podstawowe siły poruszają się w ok. Na przykład siła elektromagnetyczna rozchodzi się z taką samą prędkością jak fale EM, tj. C. Dla uproszczenia przyjmijmy, że wszystkie podstawowe siły propagują się w c jako właściwość na (1) Dlatego nie mogą one zwiększyć prędkości żadnego ciała materialnego, które już porusza się z prędkością od c do> c.

Uwaga że siły muszą przemieszczać się w przestrzeni szybciej niż c, aby spowodować prędkość większą niż c. Ale wiemy, że poruszają się w c. Więc nawet podstawowe siły stają się nieskuteczne dla ciała, które już porusza się w c.

Siły stają się nieskuteczne w kierunku ruchu ciała w punkcie c. Nadal są skuteczne w innych kierunkach, więc ciało może zostać spowolnione itp.

Dlatego prędkość propagacji sił podstawowych jest właściwością (a nie granicą). Ta właściwość ma najwyższą wartość w pustej przestrzeni, która jest c. I nic nie może rozprzestrzeniać się szybciej niż same siły - co spowoduje, że cokolwiek to zrobi? Prędkość sił (właściwość podobnie jak prędkość dźwięku) okazuje się ograniczeniem prędkości ciał materialnych. Co wcale nie jest tajemnicą.

Mogę również skomentować, że w akceleratorach cząstek wykorzystują one pola elektryczne / magnetyczne do przyspieszania cząstek i oczywiście nie mogą ich przyspieszać szybciej niż c.

Proszę LMK, jeśli to wyjaśnienie działa dla Ciebie.

A więc odpowiedź brzmi: „prędkość sił” nie jest większa niż c?Dlaczego więc następuje wzrost masy cząstek, gdy następuje wzrost prędkości?
Jak wytłumaczyłbyś siłę grawitacji działającą na foton wpadający prosto do czarnej dziury?Jeśli foton nie porusza się szybciej niż c, to czy to dlatego, że grawitacja nie wywiera na niego żadnej siły (np. Nie zyskuje masy, energii, czy nie jest przesunięty na niebiesko)?
James Watkins
2016-01-21 21:31:21 UTC
view on stackexchange narkive permalink

Jak przeczytałeś z innych odpowiedzi, nie jest to łatwe do wyjaśnienia. Wydaje się to sprzeczne z intuicją. „Jeśli chcę jechać szybciej, dlaczego po prostu nie przyspieszyć bardziej?” lub „Jeśli moja prędkość jest bardzo zbliżona do prędkości światła, wystrzelę pocisk, czy nie będzie on szybszy niż światło? Dlaczego nie?”

Względność

Zacznijmy od sprecyzowania, co rozumiemy przez „ograniczenie prędkości”. Załóżmy, że jesteś na statku kosmicznym z nieskończonym źródłem paliwa i dowolnym potencjałem przyspieszenia. Odpoczywasz (zadokowany na stacji kosmicznej) i chcesz podróżować do układu gwiezdnego oddalonego o 10 lat świetlnych. Jak długo ci to zajmie? Na tym hipotetycznym statku kosmicznym mógłbyś się tam dostać w 10 MINUT, a nie lat (zakładając, że przyspieszenie nie rozprasza twojego delikatnego ludzkiego ciała).

Ale czy to nie jest naruszenie kosmicznego ograniczenia prędkości? Nie! Technicznie rzecz biorąc, nie podróżujesz szybciej niż światło. Z twojego punktu widzenia wydaje się, że przestrzeń się spłaszcza, a cel zbliża się do ciebie. Gdyby ktoś na stacji kosmicznej patrzył, jak wchodzisz na pokład, z jego punktu widzenia podróżowałbyś bardzo blisko prędkości światła, ale zobaczyliby, że dotrzesz do celu przed upływem 10 lat.

Teraz pójdźmy o krok dalej i wyobraźmy sobie, że jesteście wiązką światła podróżującą w przestrzeni. Z twojego punktu widzenia, ile czasu zajmuje, zanim wejdziesz z czymś w interakcję? Kompletny brak czasu. Wiązka światła natychmiast teleportuje się ze źródła do celu bez upływu czasu. Ale oczywiście mają tu zastosowanie te same zasady względności - zewnętrzny obserwator nie doświadczy tej natychmiastowej teleportacji.

Brak względności

Teraz wyobraź sobie alternatywny wszechświat, w którym nie ma kosmicznego ograniczenia prędkości. Po pierwsze, światło przemieszczało się natychmiast. Kiedy więc spojrzymy w niebo, zobaczymy inne gwiazdy i planety dokładnie takie, jakie są teraz. Mogliśmy podróżować do iz dowolnego miejsca we wszechświecie w arbitralnie krótkim czasie. Wydaje się realistyczne, prawda?

Problem w tym, co dzieje się na mniejszą skalę. Wyobraź sobie proces atomowy - podobny do tego w naszym Słońcu - w tym hipotetycznym wszechświecie. Jądro Słońca ma około 15 milionów stopni Celsjusza (pamiętaj - temperatura jest związana z energią kinetyczną). Słońce ma około 4,6 sekundy świetlnej średnicy.

Relatywistyczna prędkość światła działa jak przepustnica, zapobiegając zbyt szybkiemu reakcjom łańcuchów atomowych. Pomaga ograniczyć to, jak gorące może być coś (poprzez zwiększenie masy cząstek, które poruszają się bardzo szybko, aby zapobiec ich zbyt szybkiemu przemieszczaniu się) oraz szybkość reakcji (ułamki sekundy na podróż energii w porównaniu z chwilową, która jest OGROMNA różnica). Może to oznaczać, że gwiazdy eksplodują zbyt szybko, aby się nawet uformować. Może to również oznaczać, że energia, która napędza reakcje, ucieka ze słońca zbyt szybko i nie daje mu czasu na reakcję. Nie jestem pewien, co by się stało, ale tak czy inaczej wyniki są katastrofalne.

Podsumowanie

„Kosmiczne ograniczenie prędkości” jest ważnym aspektem naszego wszechświata. Chociaż można sobie wyobrazić zaprojektowanie wszechświata bez względnego ograniczenia prędkości, wyniki nie byłyby zbyt interesujące. Nasza intuicja podpowiada nam, że powinno to być możliwe, ale czasami nasza intuicja dotycząca rzeczy, których nie w pełni rozumiemy, nie jest zbyt dobra.

Pytanie „dlaczego istnieje kosmiczne ograniczenie prędkości?” jest tak fundamentalne, jak „dlaczego wszechświat zawiera więcej materii niż antymaterii?” lub „dlaczego istnieje magnetyzm?”. Pytanie powinno zostać powtórzone: „Dlaczego żyjemy we wszechświecie o tych cechach?” Lub „Czy moglibyśmy żyć we wszechświecie o innych cechach?” Ponieważ jest możliwe, że istnieją inne wszechświaty o innych cechach i tylko niewielka ich część może faktycznie podtrzymywać życie. Jeśli życie ludzkie istnieje, będzie to naturalnie istniało we wszechświecie, który może je utrzymać.

Ten post analizuje niektóre konsekwencje ograniczenia prędkości, ale tak naprawdę nie porusza dlaczego, poza „klasycznym” to musi być w ten sposób, inaczej byśmy nie istnieli.W pewnym sensie jest to tak dobra odpowiedź, jak każda inna, jak przypuszczam.
To jak pytanie „Dlaczego Ziemia znajduje się 93 miliony mil od Słońca?”Odpowiedź brzmi: nikt nie wie dokładnie.Po prostu tak się stało.Samo pytanie jest błędne - jest całkowicie nieistotne.Prawdziwe pytanie brzmi: „Czy moglibyśmy w ogóle istnieć, aby zadać pytanie, czy Ziemia jest w jakiejś innej odległości?”
Powiedziałeś: „Gdyby ktoś na stacji kosmicznej patrzył, jak wchodzisz na pokład, z ich punktu widzenia podróżowałbyś bardzo blisko prędkości światła”, a oni nie „widzieliby”, jak wchodzisz na pokład do 10 minut przed przybyciem, prawda?W przeciwnym razie jest to absurdalne.Nie widzę światła „zaokrętowanego” aż do momentu przybycia.
@nocomprende Nie wiem, co masz na myśli.Nie mówię o wejściu na pokład światła, mówię o statku.Każdy obserwator byłby świadkiem, jak statek dociera do celu za około 10 lat, ale osoba na statku byłaby świadkiem przemijania tylko 10 minut.
„Widzielibyśmy inne gwiazdy i planety dokładnie takie, jakie są teraz. Moglibyśmy podróżować do i z dowolnego miejsca we wszechświecie bez upływu czasu”.- wtf?jak to pierwsze implikuje drugie?
@immibis: Technicznie rzecz biorąc, możesz podróżować z nieskończoną prędkością, jeśli przekonwertujesz się na dane i wyślesz dane z prędkością światła.Ale aby to zrobić, potrzebowałbyś energii proporcjonalnej do kwadratu odległości, aby odbiornik mógł faktycznie zrozumieć sygnał i zrekonstruować cię, więc nie byłyby to nieskończone odległości.
@MIchaelS Jeśli liczy się to jako „podróż”, co jest kontrowersyjne.Jednak z całą pewnością nie jest „w ruchu”.
@immibis, traktujesz ten akapit zbyt dosłownie.Staram się przekazać intuicyjne wyjaśnienie, jak wyglądałby wszechświat bez ograniczenia prędkości.Jeśli nie ma ograniczenia prędkości, możesz podróżować dowolnie szybko, co oznacza, że możesz pokonać dowolną odległość w arbitralnie krótkim czasie.
Proton przy 15 milionach kelwinów nie jest relatywistyczny.Wydaje się, że nie ma podstaw do wybuchu słońca, jeśli porzucimy ideę szczególnej teorii względności i pozwolimy na działanie na odległość.
@LLlAMnYP Obliczenia z tyłu obwiedni sugerują, że atomy poruszają się z prędkością około 0,2% prędkości światła słonecznego, przy około .0002% dylatacji czasu.I oczywiście promieniowanie elektromagnetyczne rozchodzi się natychmiast.Z pewnością byłoby inaczej, ale zgadzam się, że wynikiem może nie być eksplozja.
Prędkość światła i niemożność przekroczenia prędkości światła niekoniecznie muszą być tym samym.Dlaczego prędkość światła nie mogłaby być taka, jaka jest, ale w statkach kosmicznych można podróżować szybciej?Weźmy jako przykład prędkość dźwięku i naszą zdolność do jej przekraczania.
@Neil Są tym samym.Nie powinniśmy nawet nazywać tego „prędkością światła”, tak naprawdę powinniśmy nazywać to „względnym ograniczeniem prędkości”, a jedną z właściwości światła jest to, że porusza się z tą prędkością.I znowu możesz podróżować szybciej niż ten limit, ale tylko z pewnego punktu widzenia.
@JamesWatkins To argument antropiczny.To tak, jakbyś zapytał, dlaczego niebo jest niebieskie, i odpowiesz: „Ponieważ tylko tak może być”.W tym przypadku może to być nawet właściwa odpowiedź, ale nie rozumiesz.Może lepsze pytanie brzmi: „* Dlaczego * nie mogą być oddzielnymi koncepcjami?”
@Neil Mogliby być.Nie ma reguły, która mówi, że „nie można mieć wszechświata bez ograniczenia prędkości”.Zatem ogólne pytanie brzmi: „Dlaczego nasz wszechświat ma taki?”I to jest tak fundamentalne, jak pytanie „Dlaczego nasz wszechświat ma magnetyzm?”Poza ciężką matematyką nie ma satysfakcjonującego wyjaśnienia.
John McNamara
2016-01-22 03:58:44 UTC
view on stackexchange narkive permalink

W przeciwieństwie do innych odpowiedzi postaram się udzielić prostej odpowiedzi.

Po pierwsze, miej świadomość, że „dlaczego” jest kiepskim pytaniem dla współczesnej nauki, ponieważ współczesna nauka woli przewidywać „co” wydarzy się tak dokładnie, jak to możliwe, przy użyciu „modeli” tego, co ich zdaniem robi rzeczywistość.

Prędkość i czas są ze sobą silnie powiązane i faktycznie podlegają temu samemu „limitowi prędkości”. Kiedy poruszasz się tak wolno, jak to możliwe w przestrzeni, lecisz tak szybko, jak to możliwe w czasie i na odwrót.

Nasz obserwowalny wszechświat wydaje się mieć uniwersalne ograniczenie „czasoprzestrzeni”. część połączonych ze sobą „ podstawowych stałych” naszego wszechświata.

Interesujące jest to, że gdyby któryś z nich został zmieniony w jakikolwiek znaczący sposób, nasze obecne modele przewidują bardzo różne wersje rzeczywistości w którym jest bardzo mało prawdopodobne, że powstanie świadome życie (proszę wybaczyć dzikie spekulacje), aby móc zadać to pytanie.

Nazywa się to zasadą antropiczną

A więc odpowiedź na twoje pytanie

Czy wiemy DLACZEGO w naszym wszechświecie obowiązuje ograniczenie prędkości?

brzmi „tak, ponieważ mamy szczęście żyć we wszechświecie, którego pozornie przypadkowe ograniczenie prędkości pozwoliło na pojawienie się świadomego życia "

Może istnieć wiele takich wszechświatów, być może z innymi podstawowymi stałymi niż nasz, być może z radykalnie różne formy czującego życia.

Prawdziwe pytania FIZYKI stojące za całą tą filozofią obejmują

  • „Czy istnieją inne wszechświaty?”
  • „Co można wiemy o nich? ”
  • „ Jakie są rozkłady prawdopodobieństwa podstawowych stałych w tych wszechświatach? ”
  • itd. itd. itd.

Niestety nie Czy sądzisz, że nie zaproponowano jeszcze żadnych praktycznych eksperymentów naukowych w celu przetestowania tych modeli?

Zatem ten temat jest bardziej filozofią niż fizyką , więc twoje pytanie powinno być prawdopodobnie zamknięte -topic?

Sformułowanie zaktualizowane w odpowiedzi na komentarz.

FYI, zasadą antropiczną są spekulacje.Jest używany jako dodatkowy czynnik w celu zmniejszenia zoo, w którym znalazła się teoria strun, bez dobrego rozwiązania.Gdyby wszechświat był inny, życie byłoby inne, proste, bez powodu, by czuć się szczęśliwszym niż to konieczne.Zwróć uwagę na różnicę między * różnymi formami życia * a * brakiem życia *
Niektórzy twierdzą, że zasada antropiczna jest po prostu sformułowana: „Wszechświat jest taki a taki, więc kosmolodzy mogą go obserwować”.Nie jest to dobre dla moich standardów
Zasada antropiczna nie mówi, że „jest to jedyne możliwe życie we wszechświecie, w którym mogłoby ewoluować”.Mówi po prostu, że „gdyby ten wszechświat nie mógł wyewoluować życia, nie byłoby tu nikogo, kto zapytałby, dlaczego nie”.To argument przeciwko „wszechświat został specjalnie dla nas zaprojektowany”, a nie przeciwko istnieniu alternatywnych wszechświatów.
„Kiedy przemieszczasz się tak wolno, jak to tylko możliwe, w przestrzeni, lecisz tak szybko, jak to możliwe w czasie i na odwrót”.Czy jest jakieś dobrze znane równanie, które modeluje tę zależność?
@JamesWatkins Tak, równanie Lorentza opisuje dylatację czasu w kontekście szczególnej teorii względności.zobacz https://simple.wikipedia.org/wiki/Time_dilation
Duke of Sam
2016-01-22 17:56:37 UTC
view on stackexchange narkive permalink

To pytanie ma bardzo krótką odpowiedź, ale opiera się na założeniu, którego używa cała teoria względności. tj. prędkość światła jest stała dla wszystkich obserwatorów.

Opierając się na tym założeniu, trywialne jest wykazanie, że horrison zdarzenia jest obserwowany przy prędkości c podczas próby nieskończonego przyspieszenia.

Aby odpowiedzieć na pytanie, dlaczego to założenie jest słuszne, należy spojrzeć na wyprowadzenie równań Maxwella, które pokazują, że prędkość propagacji fal elektromagnetycznych jest niezależna od układu odniesienia. Wyprowadzenie tych równań jest oparte na pojęciach takich jak „zachowanie ładunku” i prawo Faradaya. Nie sądzę, aby można było odpowiedzieć głębiej na to pytanie.

tldr: Poprzez zachowanie energii => ładunek i imperialne obserwacje oddziaływań elektromagnetycznych można wykazać, że prędkość światła jest niezależna od układu inercjalnego . Można to wykorzystać do udowodnienia, że ​​istnieje uniwersalne ograniczenie prędkości wynoszące c.

Nikos M.
2016-01-22 01:07:28 UTC
view on stackexchange narkive permalink

Można powiedzieć, że to tylko wynik eksperymentalny. To światło (i inne sygnały / interakcje) nie przemieszczają się z nieskończoną prędkością / prędkością.

Można je tam zostawić i powiedzieć, że tak jest.

Można też powiedzieć: spójrz, jeśli weźmiesz tę zmienną i dokonasz tej transformacji (np. szybkość ), można ją dodać w nieskończoność, więc pytanie dotyczy prawidłowej zmiennej do użycia. Chociaż to po prostu omija rzeczywiste pytanie, zamiast go rozwiązać.

Przyjmę inne podejście i skieruję pytanie bezpośrednio do serca.

Skończona prędkość transmisji sygnału (lub interakcji) jest podstawowy wymóg utrzymania przyczynowości .

W przeciwnym razie, jeśli transmisja sygnału może być nieskończona, efekt może wyprzedzić swoją własną przyczynę , a wynikające z tego pętle przyczynowe przyczynowość przestaje być przyczynowością. Coś, co jest również faktem eksperymentalnym, ale na jeszcze bardziej podstawowym poziomie. W tym sensie jest to odpowiedź na pytanie.

Można pójść jeszcze dalej i wyprowadzić (górną granicę) skończoną transmisję prędkości bezpośrednio z rozważań termodynamicznych, co jest poza zakresem tego pytania, ale wspomnieć o tym do dalszych badań.

Jednak skończona prędkość transmisji sygnału niekoniecznie jest równoważna postulatowi Szczególnej Teorii Względności , że prędkość światła jest tą maksymalną (i stałą ) dostępna prędkość.

Można mieć wiele różnych skończonych prędkości transmisji mniejszych niż lub nawet większych niż prędkości światła w zależności od badanego procesu.

W rzeczywistości prowadzi się badania nad szybszą od światła transmisją sygnału poprzez splątanie kwantowe. Ale zostawię to w tym miejscu

Ale związek przyczynowy może wynikać z posiadania górnej granicy szybkości, z jaką informacje mogą się przemieszczać, jak powiedziałeś.Nie możesz więc użyć tego jako argumentu do wyjaśnienia, dlaczego w ogóle istnieje limit, ponieważ mówisz z perspektywy czasu
@LandosAdam, hmm jako podstawowe należy przyjąć to, co wydaje się bardziej podstawowe.Przyczynowość to bardziej głęboka i rozbudowana koncepcja, która wykracza poza dziedzinę mechaniki i prędkości.W tym sensie jest traktowany jako bardziej podstawowy.Ale mogę dygresować, jeśli odwrotność jest podawana z tą samą ogólnością :)
Ale mówię tylko o przyczynowości dla rzeczy, które dzieją się w naszym wszechświecie Z POWODU maksymalnej prędkości, a nie przyczynowości w jej ogólnym sensie.Albo inaczej mówiąc, w alternatywnym wszechświecie, w którym nie byłoby górnej granicy, nie istniałby związek przyczynowy z rzeczami takimi jak ten, o którym wspomniałeś.(Przyczynowość, chociaż w swojej ogólnej formie istniałaby)
@LandosAdam, hmm czy możesz to wyjaśnić?myślę, że nie będziesz w stanie tego zrobić.W każdym razie, jeśli zostanie pokazane, że te dwa są równoważne (lub * pojęcia izomorficzne * powiedzmy), to nie ma problemu, którego użyć.Jeśli nie, to jeden będzie bardziej ogólny niż drugi.Jak dotąd wydaje się, że to właśnie przyczynowość
W tym alternatywnym wszechświecie przyczynowość z sygnałami (z twojego własnego przykładu) nie miałaby nawet sensu, istnienia.Po prostu nie mieliby nic przeciwko skutkowi wyprzedzającemu ich własną przyczynę, ponieważ ich wszechświat i sposób, w jaki działałby, byłyby zasadniczo inne.Myślę..!Nie mogę już tego wyjaśnić, ponieważ nie jestem nawet pewien, co mówię, i mogę w tej chwili być antynaukowy (przy wszystkich założeniach)!W każdym razie rozumiem twój punkt widzenia, ale mam po prostu wrażenie, że związek przyczynowy, o którym mówisz, jest iloczynem ograniczenia prędkości, a na pewno nie na odwrót
Tak, dokładnie to też rozumiem.Większość otaczających nas struktur i interakcji jest tworzonych przez EM.Jeśli EM staje się nieskończenie szybkie, wszystko to staje się nieskończenie szybkie.I nie wiem, czy reszta sił wytrzyma tę sprawę.Czy czas jest ... elektromagnetyczny?
@LandosAdam, Rozumiem, co by się liczyło jako odpowiedź na pytanie, dlaczego ograniczone ograniczenie prędkości?Masz pomysł lub propozycję?miło by było to usłyszeć.Zauważ, że w odpowiedzi jest wspomniane, że termodynamika może wyprowadzić transmisję sygnału o skończonej prędkości bez innych założeń (może to jest lepsze? Z tej perspektywy).
Nie, nie mam pomysłu, po prostu próbuję rozgryźć pomysły ekspertów (ponieważ nie jestem jeszcze ekspertem), ale staram się to zrobić bez oszukiwania się w „wierzeniu” w każdy argument, który można przedstawić.Po prostu wydaje mi się, że wiele rzeczy, o których większość ludzi myśli, aby odpowiedzieć na moje pytanie, opiera się na istnieniu granicy, a nie na wyjaśnieniu, dlaczego istnieje (jeśli istnieje dlaczego).Przepraszam, jeśli wyjdę jako krytyczny, ale chcę po prostu dyskutować w konstruktywny sposób.
@noncom, Nie jestem pewien, czy czas (czy jest to * czas trwania *?
@LandosAdam, nie ma problemu, w końcu to jest twoje pytanie.Zobacz podobne moje pytanie [tutaj] (http://physics.stackexchange.com/questions/123525/why-should-a-physical-principle-be-applicable-to-different-systems-in-differen) (wpowiedzmy ta sama żyła).sam mam odpowiedź, ale chciałem usłyszeć, co mogą powiedzieć inni
Załóżmy, że wszechświat podążał za teorią względności Galileusza.W jaki więc sposób zostałaby naruszona przyczynowość?Podąża on już za względnością Galileusza w przybliżeniu przy małych prędkościach, ale nie naruszamy przyczynowości przy małych prędkościach.
Gdyby informacje mogły być przesyłane natychmiastowo między obiektami znajdującymi się w dowolnej odległości, w jaki sposób zniosłoby to przyczynowość?Gdybyśmy nie byli w stanie określić kolejności przyczyny i skutku ze względu na brak ładnej przerwy między nimi, czy zatrzymałoby to „przyczynę”, czy tylko zakłóciłoby nasze „postrzeganie” przyczynowości?
@Corey: Przyczynowość nie jest naruszona.Ludzie są po prostu zbyt zajęci równaniami SR, które przewidują takie naruszenia na podstawie tego, co obecnie wiemy o naszym wszechświecie.Informacje nadal nie byłyby natychmiastowe.Struktury fizyczne byłyby zbudowane z masywnych cząstek, które miałyby skończone prędkości, więc zawsze występowałoby pewne opóźnienie między transmisją, odbiorem, zrozumieniem, odpowiedzią itp. Ponadto skutek nie mógłby nigdy dotrzeć do przyczyny w żadnym momencie przed wystąpieniem przyczyny,więc przyczynowość nadal nie jest naruszana.
@MichaelS Po prostu dezorientuje mnie, gdy ludzie twierdzą, że przyczynowość rozpada się w pewnych warunkach, które nie wydają się prowadzić do żadnego konfliktu z przyczynowością.Może tak myślą o czasie?
@immibis, argument jest dyskusyjny, ponieważ względność galileusza lub nie wyklucza skończonej transmisji sygnału.Mylisz skończoną transmisję sygnału z maksymalną i stałą prędkością szczególnej teorii względności.W sensie tego postu szczególna teoria względności jest właśnie tym szczególnym przypadkiem.Nie więcej nie mniej.W rzeczywistości za pomocą termodynamiki skończone prędkości sygnału (lub prędkości charakterystyczne można wyprowadzić bez założenia szczególnej teorii względności).
@Corey, zobacz mój poprzedni komentarz.Rozróżnienie między przyczynowością a postrzeganiem przyczynowości nie jest dobrze sformułowane po prostu dlatego, że jeśli coś istnieje, to w jakiś sposób można tego doświadczyć (niekoniecznie w określony sposób).Jeśli nie ma absolutnie żadnego sposobu, w jaki można doświadczyć przyczynowości (lub jakiejkolwiek innej rzeczy), to to, czy istnieje, czy nie, jest jednym i tym samym, niczym i bez znaczenia.Jasne, mam nadzieję, że to cię nie zmyli
@MichaelS, zobacz moje poprzednie komentarze, mam nadzieję, że odpowiedzą na wszelkie zastrzeżenia, które możesz mieć
@NikosM.To analogia.Gdyby utrzymywała się teoria względności Galileusza, skończona prędkość światła nadal nie naruszyłaby przyczynowości, tak samo jak skończona prędkość gumowych piłek narusza przyczynowość.Oznaczałoby to tylko, że światło nie jest najszybszą możliwą rzeczą.
Wszyscy tutaj nawiązują do SR, kiedy skutecznie zadaje się im pytanie „a co, jeśli teoria względności Galileusza jest prawdziwa”?Ale co z tego, jeśli musimy mieć absolutny układ odniesienia, a prędkość światła nie jest taka sama dla wszystkich obserwatorów?Czy wtedy we wszechświecie coś się rozpadnie?Czy gwiazdy i życie nie powstałyby?Istnieje wiele procesów, które nie wymagają SR ani GR do ich opisu.
@LLlAMnYP tak * wszystko * by się zepsuło.Elektromagnetyzm i światło wymagałyby odpowiedników dostarczonych w zupełnie inny sposób.Posiadanie podobnych zjawisk na poziomie makro wymagałoby zupełnie innej implementacji na niskim poziomie.Szczegóły * słabej * siły, która sprawia, że gwiazdy świecą, również wyłaniają się z symetrii występujących na najniższym poziomie.Zmiana grupy symetrii czasoprzestrzeni wyciąga dywanik spod siebie.Pomyśl o próbie uruchomienia kodu maszynowego x86 na procesorze ARM!
Les Adieux
2016-01-23 06:05:05 UTC
view on stackexchange narkive permalink

Cóż, można udowodnić (teoretycznie i radzę Wam za Feigenbaum, 2008), że jednorodność i izotropia przestrzeni oraz jednorodność czasu koniecznie prowadzą do istnienia ograniczenia prędkości .

Zróbmy to: wyobraźmy sobie wzięcie Wszechświata i usunięcie (usunięcie) każdego rodzaju obiektów. Pozostajesz tylko z samą czasoprzestrzenią. W tej czasoprzestrzeni nie ma nic, bez względu na brak energii.

Czy rozsądnie jest sądzić, że w pustej czasoprzestrzeni istnieje jeden punkt uprzywilejowany od drugiego? Nie, więc puste czasoprzestrzenie są jednorodne.

Czy rozsądne jest myślenie, że w pustej czasoprzestrzeni jeden kierunek jest uprzywilejowany niż inne? Nie, więc czasoprzestrzeń również jest izotropowa.

Na podstawie tych dwóch założeń Feigenbaum wykazuje istnienie prędkości granicznej. Z drugiej strony warto również zauważyć, że w teorii Einsteina istnienie prędkości granicznej jest aksjomatem. Jest to jednak w pewnym sensie niepotrzebne. Rzeczywiście, zakładając mniej rzeczy (takich jak tylko jednorodność i izotropia), można wykazać, że musi istnieć prędkość graniczna.

Otóż fakt, że TA prędkość graniczna jest prędkością światła, jest pytaniem, które Feigenbaum stawia teoria ani nie pokazuje, ani nie dowodzi. Ten fakt musi zostać naprawiony przez eksperyment!

Ostatnie zastrzeżenie

Co tak naprawdę pozostaje prawdą w czasoprzestrzeni pełnej materii? Wszystko, co napisałem powyżej, nadal działa i jest aktualne, ale tylko lokalnie , a mianowicie na naprawdę małych obszarach czasoprzestrzeni i obszar po obszarze. powiedz, że istnieje prędkość graniczna (ponieważ samo pojęcie prędkości globalnej jest źle zdefiniowane) i może się zdarzyć, że czasami porusza się z prędkością ponadświetlną. Na przykład: odległe galaktyki oddalają się od nas z prędkością o wiele większą niż prędkość światła.

Dziękuję za odpowiedź, ale jak przeszedłeś od izotropowego i jednorodnego wszechświata do ograniczenia prędkości? (Nie do końca podążyłem za twoim argumentem)
@LandosAdam tego rodzaju wynika z teorii względności Galileusza.Kiedy zażądasz, aby fizyka była taka sama we wszystkich układach odniesienia, szybko dochodzisz do wniosku, że, powiedzmy, prędkość światła jest taka sama we wszystkich układach odniesienia, co prowadzi do niezmienniczości Lorenza i SR.Ale co by było, gdyby rzeczywiście istniał uprzywilejowany układ odniesienia?Niewiele rzeczy porusza się z relatywistycznymi prędkościami w stosunku do nas, więc nie rozumiem, dlaczego uprzywilejowany układ odniesienia miałby wiele zepsuć.
dobrze byłoby podać rzeczywiste odniesienie do artykułu Feigenbauma.Zauważ, że jednorodność i izotropia mogą być użyte do wyznaczenia ograniczenia prędkości, ale nie są one podstawowe w tym sensie, że czasoprzestrzeń bez absolutnie żadnej materii i energii może być izotropowa i jednorodna, ale niewiele znaczy.Z drugiej strony C izotropia i jednorodność materii i energii niekoniecznie muszą być zachowane. Ograniczenie prędkości można również wyprowadzić z zasady względności (uwaga, ** nie teorie względności **), którą można również wyprowadzić z przyczynowości
Dodałem link do artykułu Feigenbauma na temat arxiv.Jednak odpowiedź powinna również zawierać kluczową część matematycznego wyprowadzenia z artykułu, w przeciwnym razie jest to odpowiedź zawierająca tylko łącze.
Możesz rzucić okiem na http://arxiv.org/abs/1209.0563 i na cyfrowe dokumenty Einsteina [tutaj] (http://einsteinpapers.press.princeton.edu/vol7-trans/192?highlightText=%22neither% 20homogeneous% 22) i [tutaj] (http://einsteinpapers.press.princeton.edu/vol7-trans/156?highlightText=%22spatically%20variable%22).Przestrzeń nie jest jednorodna, a prędkość światła jest różna.
Owen
2016-01-28 05:53:42 UTC
view on stackexchange narkive permalink

Nie jest niczym niezwykłym, że systemy fizyczne mają ograniczenia prędkości.

Rozważ klasyczną wibrującą strunę elastyczną, zdefiniowaną przez równanie

$$ \ frac {\ Partial ^ 2 } {\ częściowe t ^ 2} \, y (x, t) = -a \ frac {\ części ^ 2} {\ częściowe x ^ 2} \, y (x, t) $$

Korzystając z tego równania, możesz zobaczyć, że niewielkie zakłócenie w jednej części struny rozprzestrzeni się na zewnątrz z określoną prędkością. W rzeczywistości zobaczysz, że szybkość, z jaką jakiekolwiek zakłócenie może przemieszczać się wzdłuż struny, jest ograniczona tą prędkością.

Możesz intuicyjnie wyczuć tę prędkość, wyobrażając sobie, że struna składa się z małych koralików połączonych elastycznymi nitkami, a sygnał musi rozchodzić się od kulki do koralika, co ogranicza jego prędkość.

Zobaczysz podobne zjawisko w równaniach różniczkowych dla sprężyny 3D solidne (jak kostka galaretki). Równania różniczkowe sygnału elektrycznego w przewodzie, fali elektromagnetycznej w przestrzeni lub fali dźwiękowej w powietrzu.

Prawie każdy układ, który można opisać równaniem różniczkowym, które wiąże prędkość zmiany w czasie do lokalnej właściwości, takiej jak pochodna lub gęstość, będą miały ograniczenie prędkości. Systemy fizyczne zachowują się w ten sposób dość często, ponieważ większość rzeczy na świecie składa się z mniejszych części, a makroskopowe zachowanie systemu można analizować pod kątem zachowania mniejszych części.

Zdaję sobie sprawę, że to wcale nie wyjaśnia dlaczego . Chcę tylko zaznaczyć, że posiadanie ograniczenia prędkości nie jest czymś niezwykłym ani zaskakującym.

valerio
2016-05-12 02:41:17 UTC
view on stackexchange narkive permalink

Mówiąc prościej, jest to sposób natury, aby zachować przyczynowość .Z Wikipedii:

Z drugiej strony, gdyby sygnały mogły poruszać się szybciej niż prędkość światła, naruszyłoby to przyczynowość, ponieważ pozwoliłoby na wysłanie sygnału w odstępach podobnych do kosmosu, co oznacza, że przynajmniej do niektórych inercyjnych obserwatorów sygnał podróżowałbywstecz w czasie. Z tego powodu szczególna teoria względności nie pozwala na komunikację z prędkością większą niż prędkość światła. ”

Gdyby nie było ograniczenia prędkości, możliwy byłby każdy paradoks związany z naruszeniem związku przyczynowego (może słyszałeś o paradoksie dziadka?).

Ale ta odpowiedź, podobnie jak większość innych, ma związek ze skutkiem ograniczenia, a nie jego przyczyną.
Więc nie sądzę, aby odpowiedź była możliwa.To tak, jakby zapytać „dlaczego siła jest równa masie razy przyspieszenie?” Lub „dlaczego mechaniką kwantową rządzi równanie Schroedingera?” ...
Cóż, nie sądziłem, że będzie odpowiedź.Właśnie zapytałem na wypadek, gdyby była odpowiedź, ponieważ jestem dopiero w moich pierwszych krokach jako fizyk, więc jest wiele rzeczy, których nie wiem i których nie mogę wyprowadzić (jeśli można je wyprowadzić)!:)
Lehs
2016-09-02 04:24:21 UTC
view on stackexchange narkive permalink

Fizyczna odpowiedź:

Kiedy elektrycznie naładowane ciało porusza się względem obserwatora, obserwator może zmierzyć indukowane pole magnetyczne.Energia zmagazynowana w tym polu magnetycznym dąży do nieskończoności, podczas gdy prędkość poruszającego się ciała zbliża się do c.

Nie jest to całkowicie intuicyjne w przypadku ogólnej teorii względności… Czy twierdzisz również, że dwa naładowane ciała zbliżające się do siebie w punkcie 9c będą miały większe indukowane pole magnetyczne dla stacjonarnego obserwatora obecnego w punkcie, w którym się spotykają, niż obserwuje każdy podróżnik?(Czy to tylko 2x dla obserwatora zewnętrznego, czy też wariuje, ponieważ prędkość delta między dwoma zbliżającymi się obiektami wynosi 1,8c?)
@BenPen, wszystko zależy od ruchu obserwatora.Pole magnetyczne jest różne w różnych układach bezwładności, chociaż wszystkie układy są „umieszczone w tym samym wszechświecie”.
Gary Godfrey
2016-01-28 14:37:07 UTC
view on stackexchange narkive permalink

Pewna stała c z wymiarami prędkości jest konieczna, ponieważ doładowania nie powodują dojazdów do pracy i dlatego doładowania muszą być dokonywane za pomocą bezwymiarowych (matematycznych) radianów. Stała c konwertuje prędkości na radiany. Stała c nie może być nieskończona, ponieważ spowodowałoby to dojazdy do pracy.

Nadanie obiektowi prędkości (zwiększenie) w kierunku x nie powoduje dojazdy do pracy ze zwiększeniem w kierunku y. Wzmocnienia i rotacje są empirycznie zgodne z definicją grupy. Specjalna teoria względności odkryła, że ​​wzmocnienia są członkami nieabelowej grupy Lorentza. Dla $ \ frac {v} {c} \ ll 1 $ prawdą jest, że

$$ \ operatorname {Boost} {\ left (\ frac {v_x} {c} \ right)} \, \ operatorname {Boost} {\ left (\ frac {v_y} {c} \ right)} - ​​\ operatorname {Boost} {\ left (\ frac {v_y} {c} \ right)} \, \ operatorname {Boost} {\ left (\ frac {v_x} {c} \ right)} = \ operatorname {Rotation} _ {z} {\ left (\ frac {v_x} {c} \ frac {v_y} {c} \ right)} \,. $$

Musi istnieć stała $ c $ z wymiarami prędkości, aby parametry doładowania były bezwymiarowymi radianami, tak aby ich iloczyn (kąt obrotu wokół osi $ z $) mógł również być w bezwymiarowych radianach. W porządku, że $ radian ^ 2 = radiany $, o czym świadczą wyrażenia w rozwinięciu szeregu potęg $ \ sin {\ left (\ theta \ right)} $. Obracanie wokół osi $ z $ o $ {\ left (\ frac {\ mathrm {m}} {\ mathrm {s}} \ right)} ^ {2} $ jest bezsensowne.

Gdyby $ c \ do \ infty $, to doładowania dojeżdżałyby do pracy i nie byłyby już częścią grupy Lorentza. Stała c jest podobna do stałej $ a = {\ left (\ frac {180} {\ pi} \ right)} \, \ mathrm {stopnie} $, która jest używana do konwersji kątów $ \ Theta $ ze stopni na radiany . Grupa rotacyjna (która jest podgrupą grupy Lorentza) nie jest abelowa

$$ \ small {\ operatorname {Rotation} {\ left (\ frac {\ Theta_x} {a} \ right)} \, \ operatorname {Rotation} {\ left (\ frac {\ Theta_y} {a} \ right)} - ​​\ operatorname {Rotation} {\ left (\ frac {\ Theta_y} {a} \ right)} \, \ operatorname {Rotation} {\ left (\ frac {\ Theta_x} {a} \ right)} = \ operatorname {Rotation} _z {\ left (\ frac {\ Theta_x} {a} \ frac {\ Theta_y} {a} \ right)}} \,. $$

Polecenie „$ a $ " jest konieczne. Obrót wokół osi $ z $ o $ \ left [\ text {stopni} \ right] ^ 2 $ byłby nonsensem. Gdyby $ a \ do \ infty, $ to rotacje dojeżdżałyby do pracy i nie byłyby już częścią grupy Lorentza. Gdyby rotacje zostały zmniejszone, nasz świat wyglądałby zupełnie inaczej. Nie byłoby czegoś takiego jak obrócenie obiektu o niezerowy kąt i przywrócenie mu pierwotnej orientacji. Ponadto pęd nie byłby kwantowany, a cząstki nie miałyby spinu.

Podsumowując, c (i a) są konieczne i muszą być skończone, ponieważ wzmocnienia (i obroty) są częścią nieabelowego Lorentza Grupa. Z tej grupy pochodzi parametr doładowania $ \ left [\ text {rapidity} \ right] = \ tanh ^ {- 1} {\ left (\ frac {v} {c} \ right)} $ w odpowiedzi Kostyi.

Niezłe podejście do tematu, ale pośrednio zakłada konkluzję przed osiągnięciem go.Przyjmowanie symetrii czasoprzestrzennych w celu zaspokojenia grupy Lorentza sprowadza się do zależności prędkości światła ...
Jesteśmy przyzwyczajeni do historycznej ścieżki do SR, w której niezmienność symetrii czasoprzestrzeni metryki Minkowskiego została odkryta jako pierwsza, a c było potrzebne do nadania ct tego samego wymiaru co x.O tym, że te transformacje symetrii tworzą grupę Lorentza, dowiedzieliśmy się później, jako wywodzące się z tej symetrii.
... Załóżmy jednak, że po raz pierwszy odkryliśmy empirycznie, że efektem doładowania nie jest tylko dodanie prędkości, ale spowodowanie pewnej rotacji, a przyspieszenie nie powoduje dojazdów.Widząc, jak zachowują się produkty, odkrywamy, że możemy zidentyfikować elementy grupowe O (3,1) z obrotami i wzmocnieniami oraz z powodzeniem sprawić, że nasze symbole na kartce papieru skopiują obserwacje świata rzeczywistego.Dopiero wtedy widzimy, że element liniowy $ ds ^ 2 = dx ^ 2 + dy ^ 2 + dz ^ 2- (cdt) ^ 2 $ pozostaje niezmieniony przez grupę.Teraz możemy powiedzieć, że grupa jest podstawowym powodem, dla którego potrzebne jest c.Niezmienność symetrii metryki jest wówczas wynikiem wtórnym.
Grupa czego?... o transformacjach przestrzennych i / lub czasowych, które odwzorowują układy fizyczne na układy fizyczne izomorficznie (w odniesieniu do fizyki).To właśnie miałem na myśli przez symetrie czasoprzestrzenne.Otóż, moim początkowym punktem jest to, że założenie (lub obserwacja) tej struktury bezpośrednio implikuje stałość prędkości światła.Pytanie o ograniczenie prędkości brzmi teraz, dlaczego ta szczególna struktura grupowa przejawia się w rzeczywistości.
joseph f. johnson
2016-02-12 00:25:08 UTC
view on stackexchange narkive permalink

W GR prędkość światła nie jest stała, zmienia się wraz z krzywizną czasoprzestrzeni. Zatem stałość tej uniwersalnej prędkości zależy od tego, czy przestrzeń - czas ma stałą krzywiznę. Co tak nie jest, ale jest to lokalnie przydatne przybliżenie i aby odpowiedzieć na zamiar PO, będziemy odtąd zakładać, że Wszechświat jest przestrzenią o stałej krzywizny. W każdym razie wiemy, że prędkość światła wynosi wolniej, gdy krzywizna jest większa, więc jeśli szukamy granicy, musimy rozważyć przypadek stałej zerowej krzywizny, ponieważ wszędzie indziej będzie wolniejsza.

Teraz dla uproszczenia załóżmy tę krzywiznę wynosi zero.

Doświadczalnie zaobserwowano, że masa jest równoważna energii, więc mają te same jednostki. Ale dodatkowa masa wytworzona przez energię kinetyczną Veolcity v wynosi $ {1 \ over 2} m v ^ 2 $, więc v musi być bezwymiarowe. Dlatego istnieje układ współrzędnych czasoprzestrzeni, w którym współrzędne x mają takie same jednostki jak współrzędne t. Ponieważ rozmaitość jest całkowicie płaska, praktycznie euklidesowa (z wyjątkiem -1 w sygnaturze metryki), możemy wybrać układ współrzędnych, który jest, mówiąc naiwnie, wszędzie taki sam. Zatem kierunek podobny do przestrzeni można obrócić do kierunku podobnego do czasu w ten sam sposób, jednolicie, wszędzie. (To może brzmieć jak SR, ale to jeszcze nie jest SR. To jest po prostu analiza wymiarowa plus prosta geometria plus ten jeden eksperymentalny fakt dotyczący równoważności masy z energią). Ale wtedy mamy uniwersalną prędkość, taką samą konwersję między współrzędną x a współrzędną t.

Jak dotąd nie oznacza to, że prędkość jest ograniczeniem prędkości ani że ma to związek ze światłem. Ale jest kanoniczna, wewnętrzna i „fizyczna”, ponieważ zależy od stosunku konwersji masy do energii.

Następnym krokiem jest wywnioskowanie, że jest to uniwersalne ograniczenie prędkości. Odbywa się to w zwykły sposób, ponieważ przyspieszenie zwiększa masę obiektu, a więc ma zastosowanie dokładnie ilościowe „malejące zwroty”.

Więc wszystko, czego chcemy, wynika z relacji Newtona między masą, energią kinetyczną i prędkością, plus jedyny eksperymentalny fakt dotyczący równoważności masy i energii.

Uwaga: William Davidon gdzieś opublikował uwaga pokazująca, jak całość SR wynikała z równoważności masy i energii. Nie przeczytałem tego, ale sam fakt, że to zrobił, dał mi oczywiście do zrozumienia. Trzeba więc uznać ten „priorytet”.

Nie może istnieć bardzo podstawowy filozoficzny powód, dla którego masa ma być równoważna energii, ponieważ fizyka teoretyczna jest możliwa na Galilei, gdzie nie jest prawdą. Z drugiej strony, filozoficznie, zawsze można by uznać przypadek Galileusza za uwzględniony w tych ramach w tym sensie, że $ \ infty $ jest uniwersalną stałą i również uniwersalnym ograniczeniem prędkości, z równymi prawami równymi 1. (Jest równe zero jako uniwersalne ograniczenie prędkości, którego nigdy nie można zaakceptować w fizyce… nawet w teorii).

Nie [publikuj dwukrotnie tej samej odpowiedzi] (http://physics.stackexchange.com/a/235309/50583), ale zagłosuj na zamknięcie podwójnych pytań zamiast kopiowania odpowiedzi.
Drogi Józefie F.johnson.Często niezadowolone jest umieszczanie prawie identycznych [odpowiedzi] (http://physics.stackexchange.com/a/235309/2451) w podobnych postach.W takich przypadkach często lepiej jest po prostu oznaczyć / skomentować zduplikowane pytania, aby można je było zamknąć.
Mark H
2016-02-12 20:23:50 UTC
view on stackexchange narkive permalink

Zauważyłem, że im bardziej zbliża się fundamentalne teorie fizyczne, te, które opisują najbardziej podstawowe interakcje w naszym wszechświecie, tym bardziej wszystkie równania zaczynają wyglądać jak przekształcenia współrzędnych. Czasami te współrzędne znajdują się w abstrakcyjnych przestrzeniach - grupach Modelu Standardowego fizyki cząstek elementarnych i przestrzeniach Hilberta mechaniki kwantowej - ale ostatecznie fizyka jest opisem ruchu rzeczy.

Aby zlokalizować coś we wszechświecie, potrzebujesz zarówno pozycji, jak i czasu. Teraz, nawet jeśli siedzisz nieruchomo, czytając to, poruszasz się w czasie. Tempo, w jakim poruszasz się w czasie, według własnego zegarka wynosi jedną sekundę na sekundę, ale nie wszyscy się z tym zgodzą. Zobaczmy, jak szybko się poruszasz według dowolnego obserwatora.

Wyślijmy cię w podróż do układu gwiezdnego Alpha Centauri z dużą częścią prędkości światła. Mieszkaniec w miejscu docelowym obserwuje Twoją podróż i widzi, że przebyłeś odległość $ d $ (około 4 lata świetlne). Zgodnie z zegarkiem, który nosiłeś, postarzałeś się o czas $ t $, czyli mniej niż czas, w którym Alpha Centaurian zmierzył Twoją podróż z powodu dylatacji czasu. Aby obliczyć całkowitą podróż w czasoprzestrzeni, możemy połączyć podróż w dwóch wymiarach z twierdzeniem Pitagorasa: $$ x = \ sqrt {d ^ 2 + t ^ 2}. $$ Całkowity dystans, który przebyłeś w przestrzeni i czasie to $ x $; odległość, jaką przebyłeś w kosmosie to $ d $; a odległość, którą przebyłeś w czasie to $ t $ (co jest równoznaczne z określeniem wieku). Problem z tym równaniem polega na tym, że $ d $ i $ t $ są w różnych jednostkach: metrach i sekundach. Na szczęście teoria względności Einsteina dostarcza współczynnika konwersji: prędkości światła. Zatem równanie powinno brzmieć: $$ x = \ sqrt {d ^ 2 + {\ left (ct \ right)} ^ 2}. $$

Teraz pokonana odległość jest równa prędkości statku pomnożonej przez czas podróży mierzony przez Alpha Centurion (odległość mierzona w pozostałych dwóch punktach jest nazywana odległością właściwą . Właściwy czas jest mierzony przez zegar w spoczynku z mierzoną jednostką, czyli zegarkiem). Nazwijmy czas, który upłynął w Alpha Centauri $ t_ \ alpha $. $$ x = \ sqrt {{\ left (vt_ \ alpha \ right)} ^ 2 + {\ left (ct \ right)} ^ 2}. $$ Możemy powiązać $ t $ i $ t_ \ alpha $ z równaniem dylatacji czasu: $$ t = \ frac {t_ \ alpha} {\ gamma} = t_ \ alpha \ sqrt {1 - {\ left (\ frac {v } {c} \ right)} ^ 2} $$ gdzie $ \ gamma $ jest relatywistycznym czynnikiem, który pojawia się w prawie wszystkich relatywistycznych równaniach. Zauważ, że $ t_ \ alpha $ jest mniejsze niż $ t $, aby odzwierciedlić wolniejsze starzenie się szybko poruszających się obiektów (ty).

Więc teraz mamy $$ x = \ sqrt {{\ left (vt_ \ alpha \ right)} ^ 2 + \ left (ct_ \ alpha \ sqrt {1- \ left (\ frac {v} {c} \ right) ^ 2} \ right) ^ 2}. $$ Upraszczanie: $$ \ begin {align} x & = \ sqrt {v ^ 2t_ \ alpha ^ 2 + c ^ 2t_ \ alpha ^ 2 \ left (1- \ left (\ frac {v} {c} \ right) ^ 2 \ right)} \\ [5px] & = t_ \ alpha \ sqrt {v ^ 2 + c ^ 2 \ left (1- \ left (\ frac {v} {c} \ right) ^ 2 \ right)} \\ [5px] & = t_ \ alpha \ sqrt {v ^ 2 + c ^ 2 - v ^ 2} \\ [5px] & = t_ \ alpha \ sqrt {c ^ 2} \\ [5px] & = ct_ \ alpha \,. \ end {align} $$ Całkowita odległość, jaką przebyłeś w przestrzeni i czasie, jest równa prędkości światła pomnożonej przez Twój czas podróży. To prawda bez względu na prędkość. Tak więc, kiedy wspólnie poruszacie się w przestrzeni i , zawsze poruszacie się z prędkością światła! Myślenie, że różne obiekty podróżują z różnymi prędkościami, ignoruje ich ruch w czasie. Tak więc prędkość światła to nie tylko prędkość maksymalna. To także minimalna prędkość. Można powiedzieć, że to jedyna prędkość.

Konsekwencją tego jest to, że im szybciej poruszasz się w przestrzeni, tym wolniej poruszasz się w czasie i odwrotnie. Możesz sobie wyobrazić tę sytuację, jakbyś jechał samochodem bez pedału przyspieszenia i hamulca - tylko kierownica. Porusza się zawsze z tą samą prędkością. Jeśli chcesz jechać na wschód, musisz poświęcić trochę prędkości w kierunku północnym. W ten sam sposób, jeśli chcesz poruszać się w przestrzeni, musisz poświęcić trochę prędkości w czasie. W rzeczywistości matematyka działa tak samo dla względności, jeśli wyobrazisz sobie jedną oś jako przestrzeń, a drugą jako czas, tak jak to zrobiłem w powyższym wyprowadzeniu.

Czy istnieje podstawowy powód, aby odpowiedzieć dlaczego tak jest? Najlepsze, co mogę wymyślić, to zauważyć, że nie mamy kontroli nad tempem starzenia się. Jeden rok dla ciebie to dokładnie ten sam rok dla mnie (chyba że wkrótce zostaną wynalezione jakieś zaawansowane statki kosmiczne). Gdyby czas nie różnił się tak bardzo od przestrzeni, jak się wydaje w naszym wszechświecie, to podobnie jak czas, podróż w przestrzeni również byłaby ograniczona do określonej prędkości. Podstawowym faktem dotyczącym naszego wszechświata, który wyznacza ograniczenie prędkości światła, jest współzależność ruchu w przestrzeni i czasie.

Farcher
2016-02-13 15:21:09 UTC
view on stackexchange narkive permalink

Pytanie w nagłówku brzmiało

Czy wiemy, dlaczego w naszym wszechświecie obowiązuje ograniczenie prędkości? ”

Potem nastąpiło wzmocnienie

To pytanie dotyczy tego, dlaczego mamy uniwersalne ograniczenie prędkości (prędkość światła w próżni). Czy istnieje bardziej fundamentalne prawo, które mówi nam, dlaczego tak jest? Nie pytam, dlaczego ograniczenie prędkości jest równe c, a nie coś innego, ale dlaczego w ogóle istnieje ograniczenie.

Myślę, że ze wszystkich odpowiedzi @Anna_v jest najbliższa do odpowiedzi na pytanie.

Obecnie odpowiedź na pytanie brzmi „nie”.

Idea , że istnieje uniwersalne ograniczenie prędkości, pochodzi z obserwacji Wszechświata.
Te obserwacje prowadzą do pewnych teorii, które można wykorzystać do prognozowania Wszechświata.
Wiele z tych teorii, które są dobre w prognozowaniu, zakłada pomysł , że istnieje uniwersalne ograniczenie prędkości.
Innymi słowy, uniwersalne ograniczenie prędkości jest użytecznym postulatem, ponieważ sprawia, że niektóre z teorii „działają”.
Obecne teorie nie są w stanie przewidzieć wszystkiego, co się wydarzyło i co się wydarzy, dlatego naukowcy szukają lepszych teorii.

Czy istnieje teoria, która wyjaśni dlaczego istnieje uniwersalne ograniczenie prędkości, nie jest znane.

Obecnie taka teoria nie istnieje.

Sean
2016-08-30 21:34:13 UTC
view on stackexchange narkive permalink

OK, mówimy o prędkościach i fakcie, że istnieje ograniczenie prędkości. Dlatego mówimy o ruchu. Ruch zawiera dwie zmienne. Jedna to prędkość, a druga to odległość. Zmienne mieszczą się w zakresie od zera do nieskończoności. Zatem, aby spojrzeć na możliwie największy obraz dotyczący ruchu, należałoby naturalnie przesunąć obie te zmienne do nieskończoności.

Podróżować z nieskończoną „prędkością”, to znaczy podróżować na dowolną odległość w zerowym czasie. Oznacza to, że można podróżować z punktu A do punktu B w powiedzmy 1 minucie, ale oznacza to również, że można podróżować szybciej i zakończyć podróż w mniej niż powiedzmy 1 sekundę. Im szybciej jedziesz, tym mniej czasu potrzeba. To są ograniczone prędkości. Ale jeśli w mgnieniu oka przemieścisz się z punktu A do punktu B, nie ma sposobu, aby pokonać tę konkretną prędkość. To jest nieskończona prędkość.

Dalej. Przemierzanie nieskończonej „odległości” oznacza, że ​​będziesz szedł w nieskończoność, ponieważ nieskończona odległość nie ma końca. Tak więc, jeśli połączysz te dwa elementy i przemierzysz nieskończoną odległość z nieskończoną prędkością, oznacza to, że będzie trwać wiecznie, w mgnieniu oka . Holistycznie jest to po prostu niemożliwe. Jednak relatywistycznie jest to możliwe i jest to możliwe, ponieważ te dwie skrajności rozdzielają się.

W jednej z możliwych skrajności możesz poruszać się w przestrzeni, ale nie będziesz się poruszać w czasie. W drugiej możliwej skrajności będziesz poruszał się w czasie, ale nie w przestrzeni. Aby to przejście było możliwe, jeśli zyskujesz w jednym, musisz przegrać w drugim. Dlatego nie możesz mieć obu skrajności jednocześnie. Zatem one z kolei muszą zostać objęte skończonością, aby spowodować to zjawisko zysku i straty.

Z kolei możesz poruszać się w przestrzeni, ale nie w czasie, podczas gdy dla obserwujących cię czas dla nich wciąż tyka, a zatem nawet jeśli czas stoi w miejscu, czas może trwać wiecznie gdzie indziej. W ten sposób możesz trwać wiecznie, w mgnieniu oka.

Zatem skończony ruch w czasoprzestrzeni jest warunkiem umożliwiającym ruch.Zatem istnieje skończona granica prędkości ruchu w przestrzeni.

Jeśli następnie przeanalizujesz wynik tego zjawiska, niezależnie odkryjesz Szczególną Teorię Względności i niezależnie wyprowadzisz wszystkie jej równania matematyczne.Obejrzyj ten film do weryfikacji.

W przypadku głosowania przeciw proszę podać wyjaśnienie.Przecież moje logiczne myślenie dotyczące prostej analizy „Ruchu” doprowadziło mnie do niezależnego odkrycia zjawiska SR i niezależnego wyprowadzenia równań SR, i zrobiłem to w sposób, o którym nikt inny nie pomyślał.póki co.Jeśli moja logika jest błędna, to nie ma ograniczenia prędkości, a równania SR również są błędne.To prawda, nie otrzymałem żadnego wcześniejszego wykształcenia fizycznego.Ale nauczyłem się, że jeśli sam odkryjesz SR, będziesz uważany za osobę zacofaną.Może to dotyczyć pojawiających się głosów negatywnych.
Nie jestem zwolennikiem przegranej, ale wydaje mi się, że ogólny pomysł jest taki, że twoje wyjaśnienie jest niejasne (nie jest wystarczająco rygorystyczne, aby wyciągnąć jakiekolwiek roszczenie).Oto dlaczego: powiedzenie, że „[Jeśli] podróżujesz po nieskończonej odległości z nieskończoną prędkością, oznacza to, że będziesz szedł wiecznie, w mgnieniu oka”. Jest fałszywe.Anulujesz nieskończoności bez określania asymptotycznego zachowania twoich wielkości.Sprowadza się to do stwierdzenia $ 0 \ cdot \ infty = \ infty $, co jest niezdefiniowane.Po drugie, pośrednio zakładasz konkluzję, stwierdzając swoje dwie skrajności, już poświęcając trochę czasu, aby zbliżyć się do wymiaru przestrzennego.
G. Bergeron - Cóż, zawsze możesz obejrzeć moje filmy na YT i zobaczyć, jaki jest wynik.Są dostępne poprzez mój profil sieciowy.
Cristian Dumitrescu
2019-12-23 13:48:49 UTC
view on stackexchange narkive permalink

Dlaczego w naszym wszechświecie obowiązuje ograniczenie prędkości?Może to mieć coś wspólnego z zasadą lokalności w fizyce.Zauważ, że we wszechświecie Fredkina jako automatu komórkowego zawsze istnieje ograniczenie prędkości dla każdego pojawiającego się wzorca (tylko przykład).Zatem istnienie ograniczenia prędkości w naszym wszechświecie jest potwierdzeniem (konsekwencją) zasady lokalności w fizyce.Na marginesie, argumenty kwantowej nielokalności (oparte na eksperymentach ze splątaniem kwantowym) muszą być wyjaśnione w kategoriach zsynchronizowanych systemów chaotycznych, ale nie odrzucając zasady lokalności.

Charles Francis
2020-05-21 10:37:37 UTC
view on stackexchange narkive permalink

Punktem wyjścia dla argumentu Einsteina było to, że współrzędne czasoprzestrzeni nie są fizycznym wyprzedzeniem, ale zostały ustalone na podstawie procedur pomiaru fizycznego. Jeśli przyjmiemy ogólną zasadę względności

  • Lokalne prawa fizyki są takie same niezależnie od materii odniesienia, której dany obserwator używa do ich ilościowego określenia.

wtedy wszyscy obserwatorzy ustawiają współrzędne w ten sam sposób, co będzie oznaczało, że maksymalna prędkość (jeśli istnieje) będzie taka sama dla wszystkich obserwatorów. Albo w naturze jest maksymalna prędkość, albo jej nie ma. Pomijając argument, że brak maksymalnej prędkości jest sprzeczny z obserwacją, możemy zauważyć, że wszystkie procesy fizyczne wymagają czasu. Brak maksymalnej prędkości oznaczałby możliwość natychmiastowego działania na odległość (przynajmniej w pewnej granicy), którą Newton opisał jako

tak wielki absurd, że uważam, że żaden człowiek, który ma w sprawach filozoficznych kompetentny wydział myślenia, nie może się w to kiedykolwiek dostać .

Yogi DMT
2016-09-21 01:35:37 UTC
view on stackexchange narkive permalink

Ilość masy jaką ma obiekt = jak odporny na zmiany pędu jest ten obiekt.Im więcej masy, tym więcej energii potrzeba do zmiany pędu.Ponieważ fotony są bezmasowe, nie mają odporności na zmiany pędu.Fotony to zasadniczo prędkość, z jaką energia rozchodzi się we wszechświecie.A jeśli chodzi o to, dlaczego jest c, tak właśnie się składa wszechświat.Nie ma prawa, że coś nie może poruszać się szybciej, ale jeśli weźmiemy pod uwagę świat fizyczny jako dwoistość energii / masy, to nic fizycznego nie może poruszać się szybciej niż światło.

Guill
2016-11-12 12:44:07 UTC
view on stackexchange narkive permalink

Powodem, dla którego w naszym Wszechświecie jest ograniczenie prędkości, jest to, że nie mamy nic (żadnej energii / siły), co mogłoby pomóc nam poruszać się czymś szybciej niż najwyższa dostępna prędkość $ \ left (c \ right) $.

Innymi słowy, jeśli prędkość $ c $ wynosiłaby $ 2 {\ cdot} {10} ^ {8} \ frac {\ mathrm {m}} {\ mathrm {s}} $ (lub 4 $ {\ cdot} {10} ^ {8} \ frac {\ mathrm {m.}} {\ Mathrm {s}} $), to this będzie ograniczeniem prędkości Wszechświata.Najlepsze, co mogliśmy zrobić, to match z prędkością $ c $.

Wookie
2019-12-16 02:02:21 UTC
view on stackexchange narkive permalink

Światło porusza się we wszechświecie najszybciej ze względu na swoją prędkość, ale także ze względu na swoją drogę.Pierre de Fermat utrzymywał, że „światło przemieszcza się między dwoma punktami na najkrótszym czasie”.Można to nazwać zasadą najmniejszego czasu.Następnie jest zasada najmniejszego działania.Cytując Pierre'a Louisa Maupertuisa, „natura jest oszczędna we wszystkich swoich działaniach”.Czy możesz pomyśleć o naturalnym przykładzie, w którym marnotrawione są zasoby / energia?Ostrożnie zasugerowałbym, że natura nie potrzebuje ruchu szybszego niż światło i dlatego go nie zapewniła.

Głos „przeciw” oznacza, że odpowiedź jest prawidłowa (w tym przypadku)
BenPen
2016-09-01 21:39:08 UTC
view on stackexchange narkive permalink

Prędkość światła dotyczy czegoś więcej niż tylko światła, nic nie może się szybciej rozprzestrzeniać, w tym siły działające na materię. Istnieje jednak nietradycyjna odpowiedź, dlaczego. Prędkość światła jest prędkością maksymalną, ponieważ wszyscy podróżujemy w nośniku fali, czasoprzestrzeni. Światło, jako fala w ośrodku, porusza się w nim ze stałą prędkością; i jak wszystko inne w płynnej podróży w „płynie”, poruszamy się wolniej niż fala przemieszczająca się w płynie czasoprzestrzeni, jak obserwujemy w przypadku obiektów w wodzie lub powietrzu lub fal w ciałach stałych. (Ciała stałe są przypadkiem szczególnym, ponieważ przechodzą przez nie tylko fale).

Analogia wskazuje, że może nie istnieć bezwzględna granica prędkości , ale wszystko, co zaobserwowaliśmy, przemieszcza się przez „płyn” czasoprzestrzeni i nie może być użyte do przyspieszenia poza prędkość światła, więc utknęliśmy. Istnieją klasycznie dwa przypadki przyspieszenia.

1) To, co przyspiesza, ma masę:

Zauważyliśmy, że masa obserwowana przez zewnętrznego obserwatora zmienia się jako stosunek $$ \ frac {1} {\ sqrt {1 - v ^ 2 / c ^ 2}} $$ Zatem z naszych obserwacji wynika, że ​​masa zmierza do nieskończoności, gdy v zbliża się do c , ale nie wiemy dokładnie, „dlaczego”, ale działa to jak zwiększający się opór podczas podróży poprzez „medium” lub malejące powroty, próbując nadać pędowi przemieszczający się obiekt. W zależności od energii wymaganej do wzrostu prędkości, twój przyrost prędkości maleje, ponieważ opór ruchu, „masa”, staje się coraz większy i ostatecznie, z powodu „masy”, energii (która jest falami lub materią na tym samym medium, co obiekt, który przyspieszamy, więc musi poruszać się wolniej niż prędkość światła) w końcu staje się tak nieefektywny, że nie powoduje efektu dla żadnego znanego źródła energii.

2) To, co przyspiesza, nie ma masy:

Jedyną rzeczą, o której wiemy obecnie, która nie ma masy, ale jest w jakiś sposób mierzalna, jest fala / cząstka, która nie może przyspieszyć, po prostu przemieszcza się przez płyn czasoprzestrzeni.Grawitacja, EM jak światło, podstawowe siły rozchodzą się jako fala na nośniku.I znowu, te media to „czasoprzestrzeń”, ale nie wiemy, z czego są zrobione.Inne fale przechodzą przez materię i we wszystkich jest mierzalna prędkość.ALE to podróż w materii w czasoprzestrzeni.

3) Trzeci przypadek przyspieszenia materii powyżej prędkości światła jest interesującą spekulacją, ale ostatecznie daremną, ponieważ nie jest to możliwe zgodnie ze znaną fizyką, ponieważ nie ma sposobu na nadanie większego pędu po osiągnięciu prędkości światła.Gdyby ktoś dodał siłę, która pchnęła coś innego niż „przestrzeń…”, istnieje eksperyment myślowy dotyczący ruchu szybszego niż fala w ośrodku czasoprzestrzeni.

Jeśli spojrzysz na podróżowanie szybciej niż przez budzenie w innym medium, zobaczysz, że jest to możliwe, ale wszystko się zmienia. Przez chwilę myśleliśmy, że nie możemy podróżować szybciej niż dźwięk, ale ponieważ nie ma ograniczenia przyspieszenia, na granicy dzieje się coś wyjątkowego, samolot może ją przekroczyć i wywołuje za sobą stożkową falę uderzeniową. Łódź może poruszać się szybciej niż fale w wodzie i znowu, na powierzchni za łodzią pojawia się załamująca się fala, „fala uderzeniowa”. Co oznaczałoby przyspieszenie, aby podróżować szybciej niż fale podróżujące w czasoprzestrzeni? Uważamy, że nie da się tego zrobić, ale wydaje się, że w pewnym sensie wiemy, jak to może wyglądać z relatywistycznej podróży FTW o mniejszej prędkości. Istnieje nazwa fali uderzeniowej powstającej, gdy materia porusza się szybciej niż lokalna prędkość światła (typowym przykładem jest reaktor rozszczepienia), zwana promieniowaniem Czerenkowa, która śledzi z normalną szybkością przemieszczania się fali w płynie. (Widzimy światło widzialne, ładny odcień niebieskiego). W przypadku podróży w normalnych płynach, podróżowanie szybciej niż fala powoduje nowy reżim czuwania. Z prędkością światła, prędkością fali w czasoprzestrzeni, czy może być taka sama? Może, gdyby istniała jakaś inna siła, jakiś inny sposób na nadanie pędu systemowi. Coś poruszającego się szybciej niż c mogłoby potencjalnie utworzyć falę uderzeniową i być może w nowy sposób odkształcić czasoprzestrzeń. Istnieją wskazówki, że czasoprzestrzeń działa jak płyn ściśliwy masowo; Jeśli wykluczysz grawitację jako siłę, która może oddziaływać bezpośrednio na światło, światło zakrzywiające się wokół wystarczająco dużej masy wygląda jak to, co dzieje się, gdy prędkość obiektu zmienia się w kierunku prostopadłym do jego ruchu, co nazywamy załamaniem. Prawdopodobnie jest to jednak błędne porównanie, ponieważ nie ma dowodów na to, że światło MOŻE przyspieszyć; w przeciwnym razie horyzont zdarzeń wokół czarnej dziury nie pochłonie światła. Ale są ludzie, którzy patrzą na gęstość przestrzeni. Tak więc są badania na krawędziach, ale przekroczenie granicy c Nie jest takie prawdopodobne, ale interesujące do rozważenia.

MrFrety
2017-06-08 21:00:32 UTC
view on stackexchange narkive permalink

Nie zgadzam się co do tego, że w naszym wszechświecie istnieje uniwersalne ograniczenie prędkości, chyba że oczywiście zdefiniujesz słowo „prędkość” w bardzo nieintuicyjny sposób.

Twierdzę, że podzielenie właściwej długości przebytej odległości w klatce obserwatora przez właściwy czas w klatce obiektu $ \ left (= \ gamma \ cdot v \ right) $ jest bliższe naszemu intuicyjnemu zrozumieniu prędkości niż zmienna $v $ (który jest zdefiniowany w odniesieniu do nieintuicyjnej definicji czasu w szczególnej teorii względności).Ta ilość nie jest ograniczona przez $ c $.

Równoważnie, można wziąć wielkość wektora składającego się tylko z przestrzennych składowych czterech prędkości.Ponownie, to nie jest równe $ v $ i może być większe niż $ c $.

Jestem zdezorientowany: wielkość kwadratowa przestrzennej części wektora prędkości to $ \ gamma ^ 2 v ^ 2 = v ^ 2 / (1-v ^ 2 / c ^ 2) $.Jeśli $ v> c $, jak sugerujesz, wielkość ta staje się ujemna, co nie ma sensu dla kwadratu wielkości tego wektora.
Jakie masz dowody na to, że $ \ mathrm dv / \ mathrm d \ tau $ jest nieograniczone?
@ZeroTheHero To nie v staje się większe niż c, ale razy gamma v.
(Które możesz łatwo sprawdzić samemu @KyleKanos.)
@MrFrety: Nie jestem pewien, co masz na myśli, jak mogę to sprawdzić samodzielnie?
@KyleKanos Poczekaj chwilę, mówisz o przyspieszeniu.Zakładam, że chciałeś napisać x zamiast v. Aby rozwinąć trochę więcej: ZeroTheHero zapisał poprawnie kwadrat wielkości, o której mówię.Spróbuj - tak jak ja - wstawić kilka liczb bardzo bliskich 1 dla v do $ \ gamma \ cdot v $, ustawiając c na 1. Przekonasz się, że to wyrażenie może dowolnie przekroczyć 1 (= c).
@MrFrety: Ups, tak.Miałem na myśli $ x $.Przepraszam za zamieszanie :(
Pozwól nam [kontynuować tę dyskusję na czacie] (http://chat.stackexchange.com/rooms/60192/discussion-between-mrfrety-and-kyle-kanos).
+2 Tak, to oczywista idea, kiedy zaczynasz rozważać teorię względności.Szkoda, że ta strona nie toleruje niekonwencjonalnego myślenia, więc większość kreatywnych odpowiedzi jest mocno odrzucana.Kontynuujcie dobrą walkę.Masz teraz reputację, aby komentować w dowolnym miejscu.
Enos Oye
2017-03-03 16:09:40 UTC
view on stackexchange narkive permalink

Teoria tachionów może przynieść pewne spostrzeżenia na to pytanie. Zgodnie z teorią tachionów możliwe jest, że cząstka może poruszać się szybciej niż prędkość światła, o ile została utworzona z prędkością większą niż prędkość światła. Do prędkości granicy światła, zgodnie z teorią tachionów, można zbliżyć się od dołu lub od góry, ale nie można jej przekroczyć. Tachyon o nieskończonej prędkości będzie miał zerową relatywistyczną energię, ale będzie zyskiwał energię, gdy zwalnia do prędkości granicy światła.

$ \ hspace {100px} $

Ale tak szybsze od prędkości światła mają dziwne właściwości czasowe, cząstki przemieszczają się tak szybko, że nieustannie cofają się w czasie i zawsze będą pochodzić z przyszłości i odchodzić w przeszłość. Patrząc z naszej rzeczywistości, może to wyglądać jak cząstki nieustannie podróżujące przez tunele czasoprzestrzenne do przeszłości, a nigdy nie obserwowaliśmy takich cząstek, a nawet może nie być możliwe obserwowanie takich cząstek.

Łącze kwantowe za pomocą laserów, a splątanie kwantowe zostało zmierzone jako co najmniej 10 000 razy szybsze od światła, więc ma albo nieskończoną prędkość, albo prawie nieskończoną prędkość. Einstein nie przepadał za splątaniem kwantowym, ponieważ wydaje się, że przekracza on jego ograniczenie prędkości światła. Nie wiemy dokładnie, czym jest ogniwo kwantowe, niektórzy sugerują tunele czasoprzestrzenne, niektórzy sugerują superpozycję, niektórzy twierdzą, że jest połączony na płaszczyźnie podstawowej, fale wolne od cząstek itp., Ale jeśli połączenie kwantowe obejmuje przemieszczające się cząstki, te cząstki muszą być tachionami cząstki o nieskończonej lub bliskiej nieskończonej prędkości.

I dziwne, że w mechanice kwantowej obserwator może wpływać na wynik eksperymentu. Wynik eksperymentu cofnął się w czasie, ponieważ światło dociera do nas po kilku nanosekundach, abyśmy mogli obserwować wynik eksperymentu. Skoro obserwacja wpływa na wynik, czy obserwacja przypomina wybieranie przeszłych potencjałów kwantowych z przyszłości? I relatywnie rzecz biorąc, czy eksperyment naprawdę miał miejsce w przeszłości? Te rzeczy często mnie mylą.

Richard Feynman i Wheeler postawili dziwną hipotezę, co by było, gdyby światło było emitowane przez zaawansowany foton, który cofnął się w czasie i emituje światło. Moglibyśmy argumentować, że nie narusza to związku przyczynowego, ponieważ dla tej cząstki liniowy czas przebiega w drugą stronę. Ta odpowiedź Terry'ego Bollingera pięknie wyjaśnia tę hipotezę: Czy foton może zostać wyemitowany bez odbiornika?

Zainspirowany przez Feynmana, zastanawiam się, czy istnieje tutaj zależność, czy tachion o prawie nieskończonej prędkości i bliskiej zeru energii zostanie wyemitowany z naszego oka w tym samym momencie, w którym otrzymane jest światło, czy tachion może powrócić czas do emitera i wpłynąć na skok kwantowy, który wyemitował światło? Relacja jest zatem taka: czy nieskończona prędkość odpowiada prędkości światła w odwrotnym kierunku? Czy ktoś wie, jak to obliczyć?

Jeśli istnieje taka zależność, możemy się zastanawiać, czy nieskończona prędkość odgrywa rolę w wyznaczaniu granicy prędkości światła.



To pytanie i odpowiedź zostało automatycznie przetłumaczone z języka angielskiego.Oryginalna treść jest dostępna na stackexchange, za co dziękujemy za licencję cc by-sa 3.0, w ramach której jest rozpowszechniana.
Loading...